Sei sulla pagina 1di 35

Cardiovascular Pathology Arteriosclerosis Simply means hardening (sclerosis) of arteries May affect small arteries/arterioles (arteriolosclerosis) A not so rare

are type is fibromuscular dysplasia The big one is atherosclerosis There are other minor types Monckeberg's medial calcific sclerosis (muscular arteries of older adults around thyroid gland) Clinical Consequences of Arteriosclerosis: Ischemia of distal organs/tissues: o Myocardial infarct o Brain infarct (stroke) o Small bowel and kidney infarcts o Gangrene of lower extremities Abdominal aortic aneurysm Note: Ischemia in general is worse than hypoxia. With ischemia, tissues are hypoxic and cellular waste products are not removed. Some degree of adjustment to pure hypoxia is possible, e.g. accommodation to high altitude. From First Aid: Monckeberg calcification: Calcification in the media of the arteries, especially radial or ulnar. Usually benign; pipestem arteries. Does not obstruct blood flow; intima not involved. Arteriolosclerosis: Essential hypertension hyaline thickening of small arteries Malignant hypertension hyperplastic onion skinning Atherosclerosis: Fibrous plaques and atheromas form in intima of arteries

01/03/11

Test q: The typical histologic change in arterioles that is related to rapid and severe (malignant) elevations of BP is: Hyperplastic (onionskinning) arteriolosclerosis. (Other choices: Hyaline arteriolosclerosis, Atherosclerotic arteriolosclerosis, Marked luminal dilatation, The arterioles show no changes with hypertension.)

Fibromuscular Dysplasia The Wall Street Journal says many medical schools do not address this! Refers to focal/segmental thickening of the walls of medium to large muscular arteries (usually medial, arteries may have a string of beads appearance) o Malformative, not degenerative, process Can affect renal, splanchnic, carotid, vertebral arteries F > M, typical age 20 to 50 years Can be asymptomatic or cause hypertension, headaches, dizziness, tinnitus, even aneurysm formation with rupture or stroke if severe Condition is developmental and familial in some cases Treatment may be antihypertensive and antithrombotic meds or angioplasty if necessary Figure: Fibromuscular dysplasia String of beads appearance of the artery thickening is uneven. No specific treatment other than reduction of hypertension.

Notes: The Wall Street Journal, June 27 & 28, 2009, ran a front page story about missed diagnoses of this disorder. Missed diagnoses of rare (or fairly rare) diseases make for great lay press stories (eg. celiac disease). FMD may present with bizarre clinical manifestations depending on what organ/tissue is ischemic, leading to incorrect or non-diagnoses. th Robbins, 8 ed. has a paragraph on FMD in the blood vessel chapter and another in the kidney chapter, the latter organ being one the most commonly affected by FMD. Diagnosis is by thinking of it and obtaining imaging studies. A bruit over a large artery in the neck or abdomen may occur (although atherosclerosis is a more common cause of this). Atherosclerosis: This is the most significant type of arteriosclerosis and is responsible for more deaths in the Western world than any other disorder It primarily affects the aorta and the larger elastic and muscular arteries (the arteries that have names that you know ex: aorta) Atrophy, infarct can result Involved Arteries in Atherosclerosis: Aorta, abdominal > thoracic; worse around major ostia Popliteals, internal carotids, Circle of Willis Coronaries Usually spared are the upper extremity arteries, mesenteric & renals except at ostia. Inside of heart also spared Pulmonary arteries not involved except with severe pulmonary hypertension o Pressure in this system is much lower. If you do happen to see atherosclerosis of pulmonary arteries, indicates severe pulmonary HT Veins not involved o Exception: there is a spot where the iliac artery compresses the iliac vein may see a small amount of atherosclerotic plaque in the vein Note: If the ascending thoracic aorta is severely involved, the causes include diabetes mellitus, hyperlipidemia type II, and syphilis.
Test q: At the autopsy of an adult, a markedly dilated pulmonary arterial tree was found. The larger pulmonary arteries showed significant atherosclerosis. This finding is essentially diagnostic of: Significant pulmonary arterial hypertension. (Marfans syndrome with pulmonary artery involvement, Emphysema, Systemic hypertension, Significant atherosclerosis is never found in the pulmonary arteries.) REPEATED x2 but one year, an additional piece of info was given: She was a 48F w/scleroderma who died following a several year course of progressive dyspnea and fatigue.

Basic Lesion: The Atherosclerotic Plaque Start out small intimal plaques get larger more numerous coalesce Plaques are typically patchy and eccentric Plaques secondarily compress and thin the media Plaques consist of varying amounts of lipid and fibrous tissue Some compensatory dilatation of the vessel may occur (up to a point)
Test q: In atherosclerosis, the accumulation of fibrous tissue and lipid causes thickening of which layer of the artery? Intima (Other choices: Media, adventitia, vasa vasorum) Test q: All of these disorders primarily involve the media layer of muscular arteries EXCEPT: Atherosclerosis. (Other choices: Marfan syndrome, Monckeberg calcific sclerosis, Aortic dissection, Syphilitic aortitis)

Notes: The three principle components of the plaque are: o Cells: Modified smooth muscle cells, macrophages recruited from blood, monocytes, and T lymphocytes o Lipid: Intra- and extracellular. Mostly cholesterol and cholesterol esters o Other extracellular stuff: Collagen and elastin, proteoglycans, calcium Proportions vary in different plaques. Plaques may be mainly fibrous without much central lipid.
Test q: Which of the following most accurately describes the histopathology of an early atherosclerotic plaque? It is characterized by intimal thickening which may include foamy macrophages. (Other choices: The primary abnormality is medial smooth muscle hypertrophy, The first change observed is a clonal proliferation of endothelial cells, Calcification and ulceration are among the earliest changes noted.)

Cross-section of atherosclerotic plaque/vessel shows eccentrically-displaced lumen in contrast, arteriolosclerosis tends to be very concentric (occurs with arteriolosclerosis of transplanted organs causes failure of grafts).

Luminal surface at the top. Uncomplicated plaques are covered by a thin layer of endothelial cells. Under the endothelium is a thick layer of fibrous tissue (containing collagen fibers, whitish in color), forming fibrous cap produces raised, whitish appearance. Cap frequently overlies lipid core cornmeal-like. Comprised of extra cellular-type material: cholesterol, cholesterol esters, macrophages, mononuclear inflammatory cells, cellular debris. Dotted blue line = internal elastic membrane. Have smooth muscle (media) on the bottom. Plaque frequently compresses the underlying media so it bulges up and down as well. Leads to another complication thinning and weakening of the media. In some cases, plaques become ulcerated.

1. Normal coronary artery: Coronary artery of a baby. Normal layers and configuration. Intima single layer of endothelial cells over a very thin layer of subintimal connective tissue; almost invisible. Most of the vessel wall is the smooth muscle layer (may have mixed-in elastic fibers, depending on the vessel). Outside is adventitial tissue. In most people, there is a normal amount of intimal thickening, even in people who are not affected by atherosclerosis. 2. Atherosclerotic plaque in aortic arch with hemorrhage: 71M w/complicated atherosclerosis. Lumen of aorta is at the top. Can see fibrous cap, lipid core, somewhat thinned medial layer, adventitia below. Can make out pointy clefts in lipid core cholesterol clefts. Contain cholesterol/cholesterol ester material washed out through processing so look like empty spaces now. Can see hint of complication hemorrhage into the plaque. 3. Cholesterol clefts in atherosclerotic plaque: 71M w/complicated atherosclerosis. Acicular pointy cholesterol clefts. Not all plaques have this much lipid some have a thicker fibrous cap and small clefts.

1. Chronic inflammation in the media of an atherosclerotic plaque: 71M w/complicated atherosclerosis. Light infiltrate of mononuclear inflammatory cells (mostly T lymphocytes) commonly found in the edges of the plaque. Evidence that inflammation is involved in the atherosclerotic process. 2. Normal aorta infant: Elastic stain. Silver stain blackens elastic fibers in medial layer of arteries like the aorta. See wavy, serpiginous, parallel-arranged elastic fibers in the media nice, uniform arrangement. Makes aorta stretchy. 3. Elastic stain of atherosclerotic plaque 71M w/complicated atherosclerosis. Note thinning of media under plaque. Can see lipid core, cap. Thinning, loss of structural integrity of the media in severe atherosclerosis. Complicated Atherosclerotic Plaques: Calcification may be extensive Focal rupture or ulceration (loss of layer of endothelium on surface exposes thrombogenic layer of connective tissue) may be accompanied by: o Hemorrhage into plaque o Thrombus formation on plaque o Escape of necrotic lipid debris from plaque cholesterol emboli downstream (atheroemboli) Weakening of media aneurysm formation Note: Thrombus may occlude the lumen infarct, or may cause thromboemboli. Hemorrhage into a plaque may also cause occlusion of lumen & hematoma in vessel wall. Figures: Hemorrhage into plaque of iliac artery. 71M w/complicated atherosclerosis. Occurs in abdominal aorta > thoracic aorta. If thoracic, may indicate severe diabetes or syphilitic involvement of thoracic aorta. Can see yellowish calcifications. The Fatty Streak: Yellow, minimally raised intimal lesions seen in children & young adults Microscopically see lipid-laden macrophages in the intima streaks go away as AS takes over Location of streaks is similar (not identical) to later AS plaques Relationship to later AS is complex and unclear Notes: Both fatty streaks and AS plaques are related to blood lipids and smoking. Fatty streaks may be seen in populations where AS is not prevalent. Fatty streaks usually seen in the aorta. Worse as you get older not clinically apparent until later in life.

Figures: Fatty streaks in the aorta of an 18F In gross photo, can barely see slightly yellowish, minimally-raised streaks. In microscopic photo, note that a few small lymphocytes are present. Fatty streak is comprised of a thickened area of subintimal connective tissue without a fibrous cap. Foamy macrophages are presents in larger vessels (aorta). Basic lesion is a subendothelial collection of foamy macrophages w/o the other features of AS plaque.

Epidemiology of Atherosclerosis: Ubiquitous in N. America, Europe, Australia, New Zealand, Russia, other developed nations Much less prevalent in Central & S. America, Africa, Asia Japanese who immigrate to U.S. & adopt U.S. diet & life style acquire U.S. predisposition Notes: Age: Starts in childhood & progresses over decades. Early AS is a common autopsy finding in teens & 20s, but not clinically significant until middle age or later when AS causes symptoms/organ injury. Sex: M>F, Uncommon in women of child bearing age (unless other strong risk factors are present). The difference equals out by about age 60 70. Estrogen replacement offers some protection. Genetics: There is a well-established genetic predisposition to atherosclerosis, probably polygenic & related to the genetics of the other risk factors; i.e. diabetes, hyperlipidemia, hypertension. The Big Risk Factors: Hyperlipemia (increased LDL and decreased HDL cholesterol) Hypertension (blood pressure >140/90 mm Hg or on antihypertensive medication) Cigarette smoking st Family history of premature AS (coronary heart disease in 1 degree male relative <55 years old or 1st degree female relative <65 years old) Diabetes mellitus Hyperlipidemia: Mostly means hypercholesterolemia triglycerides are less significant Major risk due to LDL cholesterol Inverse relationship to HDL cholesterol Exercise and moderate EtOH HDL Obesity and smoking HDL Treatable by diet and pharmacologic means Notes: The significant hyperlipidemia subtypes for increased AS risk are types IIa, IIb, and III . HDL is believed to mobilize cholesterol from AS plaques and return it to the liver for excretion in the bile. See text for further evidence implicating hyperlipidemia. Pearl: Cholesterol lowering can actually cause AS plaques to regress to some extent.

Hypertension: Risk factor at all ages, but stronger risk factor if past age 45 (even if on antihypertensive meds!) Both systolic and diastolic are important Antihypertensive therapy reduces the incidence of atherosclerosis related events, especially stroke and ischemic heart disease Some evidence now that even high-normal BP increases risk (systolic 130 to 139, diastolic 85 to 89) see NEJM 345:1291-1339, 2001 Cigarette Smoking: A pack per day for several years doubles the risk of death from ischemic heart disease Cessation cuts in half Diabetes Mellitus: Induces hypercholesterolemia and markedly increases predisposition to atherosclerosis 100X increased risk of gangrene of lower extremities (rare in non-diabetics) Complex mechanisms Emerging Risk Factors: Obesity Physical inactivity Atherogenic diet Homocysteine Prothrombotic and proinflammatory factors Impaired fasting glucose

Note: Atherosclerosis may occur in the absence of any risk factors (i.e. half of heart attacks occur in persons with normal cholesterol), or some persons with risk factors may not have significant disease. Conclusions: 1. We dont know everything about the disease. 2. Genetics is important! The Metabolic Syndrome: Abdominal obesity (waist > 40 inches for men, > 35 inches for women) Atherogenic dyslipidemia ( LDL & triglyceride, small LDL particles, HDL) blood pressure, > 130/85 mm Hg Insulin resistance, fasting glucose >110 mg/dl Prothrombotic & proinflammatory states Treatment involves therapeutic lifestyle changes diet, weight reduction, increased physical activity Homocysteine: Homocysteine is known to be toxic to endothelium Children with homocystinuria may develop first MI by age 20 years Homocysteine may be mildly elevated in adult patients without the classic childhood disorder correlates with increased risk of symptomatic AS Treatment with folate, B6 or B12 may lower homocysteine, but it is controversial whether this lessens mortality from heart attack/stroke per recent large studies Notes: Homocysteine may interfere with antithrombotic and vasodilator functions (decreases availability of nitric oxide) and increases collagen production. Cystathionine synthase defect is common autosomal recessive. Babies normal at birth, later develop failure to thrive, mental retardation, ocular changes, severe increase in homocysteine levels in the blood thromboses in vascular spaces where normal ppl would never have thromboses (sinuses, IVC, lower aorta). Get precocious AS plaques. This shows that homocysteine predisposes to atherosclerosis because it is toxic to endothelium.
Test q: In homocystinuria, children may develop mental retardation along with marked, clinically significant atherosclerosis. Which of the following is true concerning adults w/elevated serum homocysteine levels? Homocysteine is toxic to endothelial cells and elevated homocysteine is an independent risk factor for atherosclerosis. (Other choices: Anyone w/an elevated serum homocysteine level has homocystinuria, Homocysteine is an acute phase reactant an indicator of systemic inflammation produced by the liver, High serum homocysteine indicates Vit E deficiency which is a risk factor for atherosclerosis.)

Inflammation/C-Reactive Protein: CRP is an acute phase reactant from the liver a nonspecific indicator of systemic inflammation o Acute phase reactants are protein-type compounds manufactured by the liver that tend to be elevated in conditions of generalized, systemic inflammation. Could go up in Crohns disease, ulcerative colitis, osteomyelitis, TB, etc. CRP levels have a strong value for predicting cardiovascular events independent of serum cholesterol Suggests that inflammation plays a major role in the pathogenesis of AS (T lymphocytes seen in plaques) Diet, weight loss, cessation of smoking all lead to CRP Statins, other cholesterol lowering drugs, and ASA all CRP There is some evidence that the strength of CRP as an independent risk factor may have been overestimated; increased WBC may also serve as a marker for systemic inflammation and is a lot cheaper Notes: Dr. Paul Ridker at of Harvard Medical School has received much attention in the lay press as well as academia in the last few years for his publications relating C-reactive protein to cardiovascular risk. See Ridker, et al., NEJM 347:1557, 2002, for an original paper, and Ridker, Circulation 107:363, 2003 for a brief review of the CRP topic. In an article in Fortune magazine in October 27, 2003, Ridkers and others discuss ideas about antiaging for the layman. A recent NEJM article on the Jupiter study provides evidence that a statin drug (Crestor) reduces the rate of cardiovascular problems in persons with CRP even if they have normal cholesterol (see The Wall Street Journal, November 10, 2008, for the business-oriented summary).

There are numerous other markers for AS risk. One recent one is cystatin. This is a marker for renal function (somewhat like creatinine). It is known that patients with renal insufficiency are at higher risk for cardiovascular events. Even low birth weight as been associated with increased risk of AS in later life!

Test q: C-reactive protein has been associated w/an increased risk of atherosclerosis-related cardiovascular events. This association is considered evidence of which of the following? Atherosclerosis is, in part, a result of chronic inflammation. (Other choices: Atherosclerotic plaques are clonal proliferations of fibrohistiocytic cells. Vitamin B6, B12, and folate may be useful in prevention of atherosclerosis. Atherosclerosis is a result of a viral or bacterial infection.) Test q: The level of serum C-reactive protein (CRP) has been shown to be directly related to the risk for an adverse cardiovascular event. Another significant factor about this relationship is: that the CRP level predicts adverse cardiovascular events independent of the cholesterol level. (Other choices: CRP is synthesized by atheromatous plaques the most plaques, the higher the CRP. Atherosclerosis is not related to inflammation. Atherosclerosis is caused by a protein secreted by the liver. A low CRP level guarantees that you will never have an adverse cardiovascular event.) Test q: The hypothesis that inflammation plays a role in the pathogenesis of atherosclerosis: is supported by elevated serum C-reactive protein as a risk factor for atherosclerosis. (Other choices: Is doubtful because of the absence of inflammation in atherosclerotic plaques, Is doubtful because of the demonstrated monoclonality of some atherosclerotic plaques, Is doubtful because treatment to reduce systemic inflammation has no effect on atherosclerosis, Is proven by the face that Chlamydia organisms can always be isolated from atherosclerotic plaques.)

Pathogenesis of Atherosclerosis: Postulate: Atherosclerosis is an inflammatory disease, and advanced lesions in the arteries are similar to endstage inflammatory changes in other tissues (e.g. cirrhosis in liver, glomerulosclerosis in kidney) AS is an ongoing process of injury and imperfect repair of the intimal layer of large blood vessels. Initial trigger is endothelial injury high cholesterol and air pollutants can damage it directly. Mechanical stress can also play a role more prominent near the ostia of large vessels where there is more mechanical stress (bifurcation of the iliacs). When endothelium is damaged, permeability increases harmful things can leak in from the bloodstream into the subendothelial connective tissue get inflammatory response. Cholesterol-containing lipids can be picked up by macrophages and oxidized toxic. Macs attract more inflammatory cells get inflammatory cycle.

Smooth muscle cells can play a role in repair of damage to a blood vessel. They migrate up, convert into facultative fibroblasts, and form the fibrous cap. Thrombus may play a role may collect, organize into fibrous tissue, and become incorporated into the plaque. Notes: Chlamydia pneumoniae has been demonstrated in plaques. Some areas of endothelium may generate more superoxide dismutase, accounting in part for the nonrandom localization of atherosclerosis. Medial smooth muscle may have a heterogeneous embryologic origin & may respond differently, accounting in part for the nonrandom localization of atherosclerosis. Inflammatory lesion is an oversimplification. AS is perhaps better described as an ongoing process of injury and imperfect repair. Could plaques be benign neoplasia or caused by an oncogenic virus? Unlikely some plaques probably are monoclonal, but probably just arise from pre-existing developmental clones. Certain viruses cause atherosclerotic plaques in chickens. Antibiotics against C. pneumoniae have reduced recurrent events in patients with ischemic heart disease. There is evidence that the fine particulate matter (soot) in diesel exhaust may act synergistically with cholesterol to activate genes that cause inflammation of blood vessels (Genome Biology 2007, 8:R149).

Vitamin E prevents oxidation of LDL, but its role in the prevention or treatment of AS is controversial. Vitamin E supplements may help and probably wont hurt since the toxicity is low (although there is some evidence that large daily doses of vitamin E are prothrombotic and linked to heart failure!). For a good, fairly comprehensive review of the pathogenesis of atherosclerosis, see NEJM 340:115, 1999.

Test q: Which of the following most closely describes the current favored theory for the pathogenesis of atherosclerosis? It is an ongoing process of injury, inflammation, and repair of the endothelium and intima. (Other choices: It is caused by a neoplasic clonal proliferation of endothelial cells, It is predominantly a medial disease that secondarily involves the intima, It is caused by repeated infections of the intima by organisms such as Chlamydia and viruses, The pathogenesis is unknown.)

1. Severe lung disease (COPD) in a 70M smoker. Alveolar structure totally destroyed. 2. Same patient, minimal atherosclerosis.

Conclusion: Atherosclerosis may occur in the absence of risk factors, or may be minimal even with risk factors!

Aneurysm: Def: An abnormal dilatation or outpouching of a blood vessel, usually the aorta (or even the heart!) Defined as a true or false aneurysm: o True: Has all layers of vessel wall (~ true diverticulum in bowel) o False: Not all layers pseudoaneurysm (~ false diverticulum in bowel) Most common causes: Atherosclerosis ( AAA) and cystic medial degeneration Notes: An extravascular hematoma or dissection of blood into the media of a vessel are examples of false aneurysms. Other Causes of Aneurysms: Syphilis (typical in thoracic aorta/arch) Infections (mycotic) o Note: Mycotic refers to any infection, bacterial or fungal. Can be true or false. Congenital defects Trauma Vasculitis (Kawasaki disease, polyarteritis nodosa) o Kawasaki = inflammatory vasculitis in children causing aneurysms of coronary arteries. For several test questions involving Kawasaki disease, see Kawasaki disease section at the end of this study guide.

3: Syphilitic aneurysm involving the aortic annulus. Heart is at the bottom of the pic. Ascending aorta rough, dilated. 4: Syphilitic aorta note tree bark appearance. Treebarking: intimal surface roughened by inflammatory process (syphilitic aortitis)

Can dilate the aorta so much that it extends down to the aortic annulus and the valve becomes grossly incompetent. Heart gets extremely large (cor bovinum) die of secondary cardiac failure, even if aneurysm does not rupture.

Abdominal Aortic Aneurysms: Occur most commonly from below the diaphragm and renal arteries to the bifurcation, & may extend into the iliacs Are true (involve all layers) Are due to weakening and thinning of the media by atherosclerosis Occurs >50 yo, M>F There is a genetic predisposition, possibly related to a weaker media Can be saccular, fusiform, cylindroid, serpiginous Can contain abundant mural thrombus Can obstruct ostia of renals, SMA, IMA, vertebrals Variants are inflammatory and mycotic Notes: AAAs are not initiated by a primary abnormality of the media. The media is secondarily weakened by severe atherosclerotic involvement of the intima. o AS is mainly an INTIMAL disease media is innocent, damaged bystander. Inflammatory AAAs feature dense periaortic fibrosis and chronic inflammation. The cause is unknown. A typical AAA can become secondarily infected (mycotic) esp. from a Salmonella gastroenteritis this may lead to rapid dilatation and rupture. 1: Atherosclerotic abdominal Aortic aneurysm, saccular shape. Aneurysm is below the branch-offs to the renal arteries see sac-like shape, may see a little dilitation of the iliacs, too. 2: Atherosclerotic abdominal aortic aneurysm with mural thrombus - Note lines of Zahn (layering effect)

Thrombus accumulates in copious amounts in saccular, outpouching part of the aneurysm (probably because flow is very turbulent here). This, combined w/AS changes in intima ton of thrombus (layering effect). Eventually, the lumen is back down to the size it was originally restores laminar flow, thrombotic process stops. Self-correcting. AAA Rupture: The dreaded complication 4 cm is the cutoff for risk of surgery less than risk of rupture (varies a little among institutions) Newer stent devices now available can be placed via intravascular route Notes: At 4 cm, about 510% per year rupture. Could possible send emboli down to the feet if surgery is performed. Can lead to tremendous retroperitoneal hemorrhage and sudden death.
Test q: A thin 70M presents w/a pulsitile midline abdominal mass. CT scan confirms an abdominal aortic aneurysm. Which of the following is the most important consideration concerning the likelihood of rupture of the aneurysm? Diameter. (Other choices: Presence or absence of mural thrombus, Presence or absence of calcification, Location of the aneurysm above or below the renal arteries.) Test q: All are true about abdominal aortic aneurysm EXCEPT: The underlying cause is usually cystic medial degeneration (myxoid degeneration of the media). (Other choices: The pathogenesis involves secondary weakening of the media due to atherosclerosis, The risk of rupture increases significantly with aneurysm diameters above 4-5cm, There is a genetic predisposition to the development of these lesions, The lesions may become secondarily infected.)

Other AAA Complications: Occlusion of a branch mesenteric/renal artery/etc. Embolization of thrombus or atheromatous material Compression of a ureter Compression/erosion of a vertebral body (can cause back pain)

Syphilitic (Luetic) Aneurysms: A complication of tertiary syphilis not common now Is due to obliterative endarteritis which weakens the media Is more typical in the thoracic aorta (not abdominal) May cause very irregular intimal scarring (tree barking) Accelerates atherosclerosis of root and arch Note: Syphilitic thoracic aortic aneurysms may cause dramatic dilation of the aorta with protrusion of the aneurysm through the thoracic inlet into the neck! Complications of Syphilitic (Luetic) Aneurysms May extend to the aortic valve ring and cause dilatation and valvular insufficiency, leading to volume overload hypertrophy of heart, sometimes massive cor bovinum Most pts with syphilitic AA die of heart failure secondary to AV incompetence
Test q: An 80M w/a history of syphilitic aortic aneurysm has massive cardiomegaly at autopsy (1300g). The most likely explanation for the cardiomegaly is: Dilation of the aortic valve ring because of the aneurysm w/resulting valvular insufficiency. (Other choices: Fat infiltration of the myocardium, Cardiac hypertrophy due to stenosis of the aortic valve, Myxomatous degeneration of the mitral valve, Cardiac amyloidosis caused by the syphilis infection) Test q: An 80F dies after a sudden onset of fever and resp failure. An autopsy was performed. In addition to bilateral bronchopneumonia, exam of the aorta shows a markedly dilated thoracic aorta. Only minimal coronary artery atherosclerosis is observed. The interior surface of the abdominal aorta shows occasional streaks of yellow atherosclerotic plaque. The luminal surface of the thoracic aorta is markedly thickened and irregular with a tree bark-like appearance. Which of the following etiologies for the aortic changes should be suspected? Syphilitic aortitis. (Other choices: Giant cell arteritis, Kawasaki disease, Marfans syndrome.)

Aortic Dissection: AKA dissecting hematoma Old term dissecting aortic aneurysm Occurs in two clinical settings: o Middle aged men with hypertension o Connective tissue disorders like Marfans (usually younger) Usually not seen with severe atherosclerosis (the extensive scarring may be protective) May occur rarely in pregnancy, or even without an obvious cause Starts as an intimal tear in the ascending aorta, then blood dissects along the media May extend proximal and distal to the intimal tear Common cause of death: rupture into a serous cavity
Test q: Risk factors for aortic dissection include all of the following EXCEPT: Bicuspid aortic valve. (Other choices: Pregnancy, Hypertension, Marfans syndrome)

Notes: The entity is not strictly an aneurysm since there is no dilatation. In Marfans, the dissection typically occurs between the middle and outer 1/3 of the media. Look at but do not memorize the DeBakey types pictured in Robbins. The dissection may involve the great head/upper extremity arteries. The dissection may also disrupt the aortic valve apparatus.

Left: Aortic dissection with an oblique intimal tear in the aortic arch. Valve leaflets of aortic valve visible at the bottom. Tears typically occur somewhere in the aortic arch ascending aorta, proximal descending aorta. Right: Classification of dissection into types A and B. Type A (proximal) involves the ascending aorta; type B (distal) does not. The serious complications predominantly occur in the region from the aortic valve through the arch.

Figure: Double barrel aorta from a chronic aortic dissection, approximately 46 YO male with hypertension. May create a new tear in the intima, emptying back into the lumen (relieving the pressure). This can give you a double-barrelled aorta. The false channel from the dissection can endothelialize and give you a false channel in parallel with the original aorta (will be irregular new lumen).

Aortic Dissection in Marfans Syndrome: Marfans syndrome is among the best known genetic causes of aortic dissection. o Autosomal dominant defect in fibrillin protein (involved in elastic fibers in aorta). The cardiac manifestations include: o Cystic medial degeneration and Ao dissection o Dilatation of Ao root o Mitral valve prolapse (floppy mitral valve) Microscopic of Ao shows fragmentation of elastic fibers and cleft-like spaces known as cystic medial degeneration
Test q: All of the following are common manifestations of Marfans syndrome EXCEPT: Atrial myxoma. (Other choices: Mitral valve prolapse, Cystic medial degeneration of the aorta, Dilation of the aortic root)

Notes: The changes are referred to as cystic medial degeneration rather than the old term cystic medial necrosis. The changes are not specific for Marfans and are also seen as an aging change. The cleft-like spaces contain amorphous extracellular material and are not true, epithelial lined cysts.

1. Cystic medial degeneration in the aorta of a 12F with Marfan syndrome note cyst-like pools of mucinous material. CMD not strictly cystic makes cyst-like spaces w/no epithelial lining. 2. Cystic medial degeneration in the aorta of a 12F with Marfan syndrome note disruption of elastic fibers (elastic stain). Broken, short, stubby elastic fibers. In areas of the medial degeneration w/the mucinous material, the elastic is entirely absent. = weakens media. Aortic Dissection Clinical Course: Typically presents with excruciating pain which progresses downward Used to have a high mortality, but now most are salvaged with aggressive control of hypertension and surgical plication of aorta May result in a "double barreled" aorta if the dissection ruptures back into the lumen of the aorta
Test q: A 73M is found dead at home alone. He has a history of coronary artery disease, renal artery stenosis w/stenting, chronic renal failure, and peripheral vascular disease. A specimen from the autopsy is shown below. The most likely cause of death is: Aortic dissection. (Other choices: Chronic renal failure, Bacterial endocarditis, Ruptured abdominal aortic aneurysm.) Could not find this picture online it is 2007 exam 2, #20.

Ischemic Heart Disease: Synonymous with atherosclerotic cardiovascular disease (90%) Divisible into 4 clinical syndromes: o Myocardial infarct o Angina pectoris (3 types) o Chronic ischemic heart disease with heart failure o Sudden death These four groups are heterogeneous and overlapping Note: Angina may be stable (the usual type), unstable (dangerous preinfarction angina), or Prinzmetal angina (atypical occurs at rest due to cor. artery vasoconstriction).
Test q: Coronary artery vasospasm is thought to be the mechanism for which of the following types of angina? Prinzmetal. (Other choices: Stable, unstable, radiating)

Pathogenesis of Atherosclerotic Cardiovascular Disease Pathogenesis involves the interplay of 4 factors: o Fixed narrowing of coronary arteries takes 75% to be hemodynamically significant o Acute plaque change Fracture/rupture/ulcer o Thrombosis o Vasoconstriction May play a role in rupture of a plaque.May play a role in rupture of plaque. Main problem of AS is ischemia of the tissues that are supplied by the affected vessels. Acute Plaque Change: Thought to trigger sudden cardiac events (and maybe unstable angina) May be due to: o Rupture or fissuring of a plaque in a small artery, may cause occlusion o Erosion/ulceration of a plaque o Hemorrhage into an atheroma May be followed by thrombus formation on the plaque surface, which may: o Totally occlude the lumen MI o Partially occlude the lumen ? unstable angina (does not predictably go away with rest), subendocardial infarct, sudden death o Embolize microinfarcts o Organize contribute to growth of plaque Notes: The most dangerous plaques are moderately stenotic (50 75%) with a relatively large, soft lipid core and a thin cap. More stenotic lesions may actually be more stable. More stenotic lesions may actually protect the myocardium against MI by preconditioning (mechanism unknown). For a good review of vulnerable plaque change, see Circulation 108:1664, 2003 Basic lesion is fibrous cap over necrotic lipid core. Not all plaques have a lot of lipid (could be mainly fibrous) amounts can vary. Relatively thin cap w/lots of lipid is more prone to rupture (sudden vasoconstriction, adrenergic stimulation, inflammation, etc. can trigger). What causes acute plaque change? Mechanisms are uncertain; may include: Adrenergic stimulation accounts for more MIs in AM or when stressed (increased MI's after earthquakes, 9/11 attacks) Vasoconstriction may contribute to plaque fracture Inflammation may play a role in destabilizing the plaque There is no diagnostic test for unstable plaque Hemorrhage into an atherosclerotic plaque in a coronary artery There may be some thrombus on the plaque.

Myocardial Infarct Cause: o ~90% due to disrupted plaque platelet adhesion occlusive thrombus o Other 10%, who knows? Prolonged, severe vasospasm Emboli from left atrium or paradoxical emboli through patent foramen ovale Abnormalities of the epicardial or intramural coronary arteries, vasculitis, hemoglobinopathies Unexplained There is always a possibility of thromboemboli from another anatomic site could have a mural thrombus in LA that gives off thrombus material embolize coronary arteries, brain, kidneys, etc.
Test q: The most common cause of MI in adults is: Disruption of an unstable atherosclerotic plaque in an epicardial coronary artery. (Other choices: Prolonged severe coronary artery vasospasm, Embolization to the coronary arteries of thrombi from other sites, Vasculitis of the coronary arteries, The cause of most MIs is unknown.) Test q: A middle-aged man collapsed and died suddenly at the office. He had no previous history of cardiac problems. Autopsy exam of the heart showed severe atherosclerosis of the coronary arteries. Other likely findings would include: Thrombosis within a coronary artery at the site of a disrupted atherosclerotic plaque. (Other choices: Coagulative necrosis of myocytes, A brisk infiltrate of neutrophils within the myocardium, An infiltrate of phagocytic cells and granulation tissue within the myocardium, Changes of pulmonary hypertension in the lungs.)

Risk Factors for MI Same as for Atherosclerosis Older age Hypertension Smoking Diabetes mellitus Hypercholesterolemia Male > female (declines progressively with age past reproductive years) Review the coronary artery anatomy: LAD supplies the apex, anterior LV wall, and anterior 2/3 of the ventricular septum. RCA Posterior LV and posterior 1/3 of septum in the right dominant heart, whole RV LCx lateral LV The left and right coronary artery systems each supply about 50% of the left ventricle in the right dominant heart. In the majority of cases, the SA and AV nodes are supplied by the RCA. In 80% of us, the RCA supplies the inferior portion of the LV via the posterior descending (PD) artery (=right dominant). In the other 20%, the PD arises from the LCx. Coronary artery occlusion most commonly occurs in the LAD, which supplies the anterior IV septum.
Test q: A posterior MI of the LV is most likely caused by occlusion of which of the following (assuming a right dominant coronary artery system and no significant collateral circulation between the major coronary perfusion territories)? Right coronary artery. (Other choices: LAD, Left circumflex, Diagonal branch from left circumflex, It is impossible to tell from this info.) Test q: Thrombosis of the posterior descending coronary artery would most likely cause infarction of what part of the heart? Posterior part of the LV and posterior third of the IV septum. (Other choices: Apex, Lateral wall of the LV, Anterior wall of the LV and the anterior 2/3 of the IV septum) Test q: All of the following numerical facts are true EXCEPT: Two thirds of us are left coronary dominant (Other choices: A diameter of around 4cm indicates that an abdominal aortic aneurysm should be repaired, At least 75% narrowing of a coronary artery is necessary to be hemodynamically significant, One third of us have a patent foramen ovale, Myocardial rupture after infarction typically occurs in the range of 3-7 days after the acute event.) REPEATED x2 but one year, the answer was Four fifths of hearts are left coronary artery dominant (i.e. the left circumflex coronary artery supplies the posterior descending coronary artery and one of the incorrect choices was modified: 4cm diameter is a typical cutoff above which surgical repair of an abdominal aortic aneurysm is indicated.

Morphology of an MI Location depends on coronary artery affected (may be fooled by well-developed collateral flow) o Predicting the involved coronary artery using the anatomic site of infarct is only ok in people who do not have a lot of compensatory anastomoses The subendocardial zone is the "end" of the blood flow o We learned about infarcts as being wedge-shaped, but in the heart they usually are not. This is because the blood flow comes from the outer (epicardial) surface inward. Subendocardial MI = inner 1/3 to , may be circumferential in the distribution of all 3 cor. arts. Transmural MI = more than half of ventricle wall thickness, usually in the distribution of one cor. art. The necrosis is coagulative (apoptosis may play a role) Notes: In MI, occlusions almost always involve one of the 3 named coronary arteries or a secondary branch (diagonal branch of LAD, marginal branch of LCx). A pure right ventricular MI is rare, but the RV may be involved by extension from LV.

Myocyte cell death occurs first in the subendocardial zone and the advances outward as a wave with an at risk zone at the leading edge. The extent of the MI depends on several factors including: o Duration of occlusion o Metabolic/O2 needs of myocardium o Hypotension, arrhythmias, spasm

1.

2.

3.

These are views of a LV cut in cross-section. You dont see the classic wedge shape because the arteries dont branch out they branch in. Creates modified wedge truncated pyramid. 1. 2. 3. Regional transmural infarction Regional non-transmural infarction Regional transmural plus diffuse subendocardial necrosis Patient had big transmural MI, which caused him to have cardiogenic shock resulted in additional subendocardial necrosis in a diffuse distribution. Diffuse (circumferential) subendocardial necrosis Diffuse focal necrosis

4.

5.

4. 5.

People in shock or people w/decreased global perfusion overall tend to develop infarcts in the end-zone, subendocardial area.

What happens to a cardiac myocyte when blood is cut off? It resorts to anaerobic glycolysis within seconds It stops contracting within about a minute (can acute heart failure) Myocyte death occurs in 20 to 40 minutes >1 hour, vascular injury occurs Gross Changes in MI 0 to 12 hours 24 hours 3 to 7 days 7 to 10 days 2 months Nothing (triphenyltetrazolium chloride stain can show in 2 3 hours) Dark mottling (trapped blood) Yellow-tan center, hyperemic margin Max. necrosis of yellow-tan center Mature scar

Light Microscopic Changes in MI 0 to 4 hours Not much wavy fibers at edge 4 to 24 hours Beginning coagulation necrosis, nuclear pyknosis, early neutrophilic infiltrate 1 to 3 days Well developed coagulative necrosis with loss of nuclei and cross striations, lots of neutrophils Up to 10 days Progressive dissolution of myocytes & ingrowth of granulation tissue 2 months Dense collagenous scar
Test q: Granulation tissue (mixed proliferating capillaries and inflammatory cells) is most prominent during which time period following an MI: 3-10 days. (Other choices: 0-4 hours, 4-24 hours, 1-3 days, greater than 8wk) Test q: A 55M is found dead at home. At autopsy, all of his coronary arteries are partially occluded by atherosclerotic plaque and the LAD coronary artery is completely occluded by a thrombus. The anterior LV is slightly hemorrhagic appearing grossly. Sections of the myocardium show swollen, red cardiac myocytes with loss of the nuclei and no significant inflammatory cell infiltrate. The probable interval between the formation of the thrombus and the time of death is: 4-24 hours. (Other choices: 0-4 hours, 1-3 days, 3-10 days.) Test q: A 60y/o previously healthy man experiences chest pain radiating to the left ear, jaw, and left arm. Evaluation of the patient is significant for abnormal electrocardiography and abnormal cardiac enzymes. Despite treatment, the patient becomes progressively hypotensive and dies 48 hours after the onset of chest pain. At autopsy, atherosclerosis is noted in the left anterior descending coronary artery w/80% occlusion of the lumen. There is a thrombus in association w/the atherosclerotic plaque. What is the expected histology of the anterior wall of the LV? Coagulative necrosis w/an early acute inflammatory cell infiltrate. (Other choices: Normal myocardium, Necrosis w/dissolution of myocytes and granulation tissue, Fibrous scar) Test q: Given the presence of coronary artery atherosclerosis in the patient in the question above, atherosclerosis would be likely in which of the following blood vessels? The ostia of the renal arteries. (Other choices: The pulmonary arteries, The arteries in the arms, The inferior vena cava) Test q: A 58M presents w/chest pain of 6hr duration. Shortly after presentation to the ER, he develops an arrhythmia, hypotension, and dies despite resuscitative efforts. At autopsy, on gross exam of the heart a large zone of darkened, mottled myocardium is seen in the anterior wall of the LV and the anterior 2/3 of the IV septum. Occlusion of which artery is most likely? Left anterior descending coronary artery. (Other: Left circumflex, Diagonal branch from left circumflex, RCA) Test q: The histologic sections of myocardium from the patient in the question above would most likely show which of the following? Swollen, red cardiac myocytes with some associated hemorrhage, but no significant inflammatory infiltrate. (Other choices: No histologic abnormality, Nectrotic myocytes w/a dense neutrophil infiltrate, A histiocyte infiltrate and granulation tissue, Fibrosis)

Test q: A 45M experiences crushing substernal chest pain after arriving at work one morning. Over the next 4hr, the pain persists and begins to radiate to his left arm. He becomes diaphoretic and short of breath but waits until the end of his 8hr shift to go to the hospital. An elevated serum value of which of the following lab tests would be most useful for the diagnosis of this patient on admission to the hospital? CK-MB fraction. (Other choices: Lipase, AST, ALT, LDH-1)

1. Triphenyltetrazolium chloride (TTC) stain showing absent staining of necrotic myocardium can show infarcted myocardium in 2-3 hours. Acute myocardial infarct, predominantly of the posterolateral LV. The staining defect is due to the enzyme leakage that follows cell death. Note the myocardial hemorrhage at one edge of the infarct that was associated with cardiac rupture, and the anterior scar (arrowhead), indicative of old infarct. Also note thick LV chamber diffuse concentric myocardial hypertrophy of the LV (may have been hypertensive for a long time). 2. Anterolateral MI, about 1 day old, in an 83 YOM with diabetes, history of stent placement in left circumflex coronary artery. Flat side = back side, diaphragmatic surface. Rounded side is anterior. Can barely start to make out the mottled appearance of MI (blood pooling, tissue hasnt really dissolved into amorphous lipid material yet). This man has tunneled a LAD covered by bridge of myocardium may or may not have clinical significance. 3. 3 day old MI, posterior and subendocardial, in a 60 YOM who died 3 days after coronary artery bypass surgery. See very well-developed yellowish coagulative necrosis. Can see hyperemic zone of dilated small blood vessels this is where the cellular apparatus for repairing the infarct comes from (from the outside edges in). Also has a little subendocardial damage of papillary muscles.

4. 3 day old MI, posterior and subendocardial, in a 60M who died 3 days after coronary artery bypass surgery. 5. Same patient as #4. Yellow color is due to lipid content amorphous soup of necrotic material. 6. 5 day old posterolateral infarct in a 63M with hyperlipidemia who received a heart transplant 9 years ago. Shrunken, shriveled, mottling around the edge of infarct.

7. Old healed MI. 6 wk-2 mo. just see whitish scar tissue. 8. Acute MI, 1 to 2 days old, note contraction bands, loss of nuclei. Subtle changes. See contraction bands in ischemic myocytes = they contract and do not relax.

1. Acute MI, 3 to 4 days old, note inflammation, hemorrhage, necrotic myocytes. Thrombosed vessels, brisk infiltrate of neutrophils 2. Acute MI at 1 to 2 weeks, organizing granulation tissue. Lots of macrophages moving in, granulation tissue briskly forming numerous small capillaries w/a myxoid connective tissue background, fibrocytes, mixture of mononuclear inflammatory cells. 3. Old healed MI, >6 weeks, scar tissue. Fibrous scar get shrinkage/contraction in wall of the heart. Reperfusion After Acute MI By thrombolytic therapy (TPA, streptokinase) or mechanical (angioplasty) May limit infarct size and improve function & survival Must be done within 3 4 hours of onset of Sx ( if done in first 20 min, may prevent all necrosis) Causes its own type of injury (reperfusion injury) o Hemorrhage due to damaged, leaky blood vessels o More rapid disintegration of critically damaged myocytes get contraction band necrosis in irreversibly injured myocytes o Some new cellular injury may occur due to O2 free radicals o Arrhythmia o Increased apoptosis of myocytes o May cause swelling of endothelial cells occlusion stops reperfusion ("no-reflow") Note: "Stunned" myocardium refers to reversible myocyte dysfunction after an ischemic event. The myocardium may take several days to recover.
Test q: If the myocardium is reperfused by thrombolytic therapy after the onset of acute MI: Capillary endothelial swelling may prevent local reperfusion to critically injured myocytes. (Other: Myocytes cannot be significantly salvaged by reperfusion bc severe ischemia causes immediate cell death, Thrombolytic therapy is usually not effective since most MIs are not caused by coronary artery thrombosis, The disintegration of myocytes lethally damaged by the preceding severe ischemia is retarded or prevented, All of the affected myocytes will be salvaged if reperfusion occurs in the range of 2-4hr after the acute event.)

Complications of MI Nearly have one or more: o LV contractile dysfunction with heart failure, hypotension, shock if >40% of LV is lost o Arrhythmias may cause sudden death (more common in LV MIs) o Pericarditis o Myocardial rupture (typical at 3 7 days) Through septum to RV Through free wall tamponade Partial rupture through free wall pseudoaneurysm filled with thrombus Papillary muscle rupture acute mitral insufficiency Infarct extension Mural thrombus may thromboembolus LV aneurysm late complication Papillary muscle dysfunction Progressive heart failure (ischemic heart disease)
Test q: The typical mechanism of death following myocardial rupture complicating MI is: Cardiac tamponade. (Other choices: Exsanguination into the pericardial and pleural spaces, Arrhythmia, Tearing of a coronary artery above the ruptured myocardium.) Test q: A widespread pattern of subendocardial necrosis in the LV may be seen: following an episode of hypotension or shock, especially in a patient with compromised coronary arteries. (Other choices: As a result of Adriamycin doxorubicin toxicity, With Chagasic myocarditis, Only with simultaneous occlusions of all three major coronary arteries, This pattern is not seen.)

Test q: A circumferential subendocardial necrosis of the LV, not corresponding to the distribution of any one coronary artery, is most likely to be seen following which of the following: An episode of hypotension or shock, especially in a patient with compromised coronary arteries. (Other choices: Administration of chemotherapy which includes Adriamycin doxorubicin, Infection by T. cruzi, Ulceration of atherosclerotic plaque in the LAD coronary artery.) Test q: A 55M presents w/chest pain of sudden onset associated w/nausea and diaphoresis (sweating). Evaluation of the patient is significant for abnormal electrocardiography and abnormal cardiac enzymes. Echocardiography shows a hypokinetic lateral wall of the left ventricle. Assuming that the patient had a lateral wall infarct, occlusion would be expected in which of the following arteries? Left circumflex coronary artery. (Other choices: LAD, RCA, Post. descending, None of the above this pattern of infarction results from marked hypotension rather than occlusion of an artery) Test q: The patient in the previous question survives the acute event, but 5 days following the onset of symptoms becomes severely hypotensive. Echocardiography demonstrates severe mitral regurgitation, not noted earlier in the patients evaluation. Which of the following complications of MI should be expected? Papillary muscle rupture. (Other choices: Rupture of the LV free wall, Pericarditis, LV aneurysm) Test q: The patient in the questions above survives this complication w/treatment and later wants to discuss cardiac risk factor modification w/the intent nd of lowering the risk of a 2 MI. Which of the following is a risk factor for MI? Elevated fasting blood sugar. (Other choices: Decreased CRP, Lower than avg LDL cholesterol, Lower than avg homocysteine)

Notes: In 1015% of pts, usually with >40% LV infarct, there is 70% mortality. Pericarditis after an MI is typically fibrinous or fibrinohemorrhagic. Typical onset 23 days after acute MI, then resolves. Large anterior transmural MIs have a higher risk of rupture, expansion, aneurysm, thrombi & in general have a worse prognosis than posterior MI's. Large posterior transmural MIs have a higher risk of heart blocks, RV infarct (or both). Compensatory remodeling occurs with healing of MI with compensatory hypertrophy and areas of thinning, scarring and dilatation. May get late decompensation of the myocardium. The conventional wisdom has been that cardiac myocytes cannot regenerate and are replaced by scar tissue when they die. There is some evidence to the contrary, but myocyte regeneration, if it occurs, does not play much of a role in the healing of an MI. Chronic Ischemic Heart Disease Results in progressive failure due to ischemic myocardial damage Usually in pts with prior MI, but may see with severe obstructive coronary artery disease without MI Hearts are typically large and heavy due to areas of compensatory hypertrophy & areas of dilatation & fibrosis Ischemic cardiomyopathy is used for this, but it is not strictly correct since it is not a "primary" disease of the myocardium 1. Massive hemopericardium at autopsy due to ruptured MI. Massive volume of blood in the pericardial sac causing tamponade (can be due to transmural infarction acute rupture of the LV). 2. Transmural infarct with rupture.

3. Left ventricular aneurysm due to old MI. (If you dont rupture all the way, can lead to aneurysm of the LV.) 4. Ruptured left ventricular papillary muscle complicating an MI. Complete rupture of a necrotic papillary muscle.

Sudden Cardiac Death Def: Rapid (<1 hr) unexpected death due to cardiac causes IHD the most common cause in older persons (acute plaque change thrombus arrhythmia) In younger persons, non-atherosclerotic causes are more common Non-atherosclerotic Causes of Cardiac Sudden Death Hypertrophic cardiomyopathy/isolated hypertrophy of unknown cause Congenital heart disease, including coronary artery anomalies Aortic stenosis (congenital or acquired) Myocarditis Pulmonary hypertension Conduction system or ion channel abnormalities (prolonged QT interval) Mitral valve prolapse (floppy mitral valve)
Test q: Aortic valvular stenosis, pulmonary hypertension, and mitral valve prolapse have all been associated with: Sudden cardiac death. (Other: LV hypertrophy, Eisenmengers syndrome, Marfans, These entities have nothing in common)

Valvular and Endocardial Heart Disease Valvular Disorders Cause: Stenosis Regurgitation (aka insufficiency) Both Murmurs Dysfunction ranging from trivial to rapidly fatal depending on how bad the defect is and how rapidly it developed May cause secondary changes in heart and blood vessels Notes: The valve leaflets are simple avascular laminated structures. The strength of the leaflet comes from the middle layer, the lamina fibrosa. A lamina spongiosa is a cushioning layer on the inflow surface. Elastic fibers are also present. The outer surfaces are covered with endothelium. The chordae tendineae are also avascular and blend with the lamina fibrosa. The arterial valves work passively. The AV valves require a functioning tensor apparatus (papillary muscles and chordae tendineae) to prevent gross incompetence. The most important secondary changes from valve damage include myocardial hypertrophy and hypertensive changes in pulmonary vasculature. Causes of Valvular Lesions May be congenital or acquired, and acquired lesions may be acute or chronic Most lesions are acquired, chronic and affect the left-sided valves Notes: Congenital causes include malformed (dysplastic) valves, which are stenotic, regurgitant, or both. Congenital valvular lesions are relatively rare. Bicuspid aortic valve is one of the most common congenital valvular abnormalities affecting 1 2% of the population. Acquired causes include degenerative diseases, infection, autoimmune disorders, and papillary muscle dysfunction (e.g. as a complication of MI). Inheritable connective tissue disorders are technically congenital, but the manifestations are acquired later in life (e.g. Marfans, floppy mitral valve). th See Table 12-7, p. 561, Robbins 8 Ed. for a listing of the major causes of acquired valvular heart disease. Most Frequent Chronic Valve Lesions of Major Functional Significance: Mitral stenosis secondary to rheumatic heart disease Mitral insufficiency secondary to myxomatous degeneration (floppy mitral valve) Aortic stenosis secondary to calcification Aortic insufficiency due to dilatation of the aortic ring secondary to hypertension or aging Calcific Aortic Stenosis This is a wear and tear lesion of aging (chronic injury due to hypertension and hyperlipidemia play a role), but not exactly the same as atherosclerosis. Typically presents in 70s & 80s (50s & 60s if bicuspid aortic valve) Hallmark is heaped up masses of calcium on the downstream side of the leaflets preventing opening of valve

Notes: Calcification begins in the fibrosa layer at the attachment of the leaflet, which is the point of maximum flexion usually does not involve free edges. The early lesion is aortic valve sclerosis. The commissures are not fused (at least early on) as in rheumatic. Also the mitral valve is not involved (although calcium may extend through aortic annulus to involve anterior leaflet of the mitral valve). Calcific Aortic Stenosis Clinical Features Progresses at a variable (slow) rate and causes an increasing pressure gradient, 75100 mmHg LV hypertrophy Angina or syncope may ensue ++ Ca may rarely extend into septum & cause AV conduction block (not mentioned in Robbins but true) Eventually get cardiac decompensation & CHF end stage with 50% mortality in three years
Test q: Risk factors for calcific aortic stenosis include which of the following? Hyperlipidemia and hypertension. (Other choices: Marfans syndrome, Cystic medial degeneration, Idiopathic inflammatory bowel disease)

Bicuspid Aortic Valve 1 2% of population The larger cusp usually has a raphe ++ Valves work OK, but are predisposed to early Ca with raphe as the main site calcific AV stenosis Bicuspid aortic valves also predisposed to infectious endocarditis Bicuspid valves associated with coarctation, aneurysm, dissection Note: A bicuspid valve may be associated w/structural abnormalities in the aortic wall. Trivia: The raphe is most commonly between what would have been the left and right cusps (80%).
Test q: All of the following concerning bicuspid aortic valve are true EXCEPT: It is only seen in association w/other congenital cardiac abnormalities. (Other choices: It is predisposed to early calcification, It is associated w/coarctation of the aorta, It can be associated w/aortic dissection) Test q: A 60M visits the physician because of worsening cough and orthopnea. On phys exam, he has dullness to percussion at the lung bases and diffuse crackles in the upper lung fields. He is afebrile. Echocardiography shows marked LV hypertrophy and severe aortic stenosis. The remaining cardiac valves are normal. A coronary artery angiogram shows no significant coronary arterial narrowing. Which of the following conditions best accounts for these findings? Bicuspid aortic valve. (Other choices: Diabetes mellitus, Marfan syndrome, Systemic hypertension, Infective endocarditis) Robbins explanation: Bicuspid valves have a tendency to calcify with aging, which eventually can result in stenosis. In individuals w/congenitally bicuspid valves, symptoms appear by 50-60 years of age. By contrast, calcific aortic stenosis of tricuspid valves manifests in the seventh or eighth decade.

Mitral Annular Calcification Refers to degenerative calcification of the mitral annulus (ring) Most common in >60 yo women, associated with floppy MV and LV pressure Usually is innocuous, but can cause regurgitation by impairing ring contraction during systole ++ Ca can extend into septum & cause AV block & even sudden death due to arrhythmia Can provide a nidus for infective endocarditis or thrombi stroke Myxomatous Degeneration of the Mitral Valve AKA Floppy Mitral Valve, Mitral Valve Prolapse The most common valvular disease in the industrialized world, affects ~ 3% of adults, mostly young women Characterized by redundant, myxoid mitral valve leaflet tissue that protrudes into left atrium on systole May see with Marfans, but cause usually unknown probably a disorder of connective tissue PE: May show late systolic click due to snapping of the chordae or scallop late systolic or holosystolic regurg. murmur Echocardiogram diagnostic Clinical: Usually asymptomatic, but may have dyspnea, fatigue, chest pain, or even psychiatric symptoms Notes: The leaflets balloon into the LA with interchordal hooding. The chordae are frequently elongated and thinned and occasionally rupture. The TV is also affected in 20 40% & sometimes even the AV and PV. Can see friction lesions where leaflets strike the LA walls or rub against each other. Can get thrombi on these surfaces. Microscopic: Attenuation of the fibrosa layer with increased myxoid tissue. Ditto in chordae. Psychiatric symptoms include depression, anxiety, and personality disorders. Strictly speaking, if MV prolapse is due to a connective tissue disorder, it should be considered a congenital valvular disorder.
Test q: A 35F reports chronic fatigue, dyspnea of exertion, and occasional chest pain. History is negative for significant cardiac or resp problems. On phys exam, a late systolic click and a systolic murmur is heard. This presentation is most compatible w/which of the following conditions? Myxomatous degeneration of the mitral valve. (Other choices: Bicuspid aortic valve, Constrictive pericarditis, Prinzmetals angina, Kawasaki disease) REPEATED x2

Test q: The histologic appearance of a mitral valve affected by myxomatous degeneration to the mitral valve is: Decreased elastic fibers and collagen and pale basophilic background substance (Other choices: A markedly thinned valve composed of uniform dense eosinophilic collagen, Necrotic fibrous tissue w/loss of cell nuclei and nuclear debris, Neovascularization and cardiac myocytes with contraction bands.) Test q: A 49F dies of conditions unrelated to cardiac disease. By history, she has had a holosystolic heart murmur w/a late systolic click. An autopsy is performed and shows an abnormal mitral valve w/redundant, floppy valve tissue ballooning back into the left atrium. Micro exam of this valve would most likely show: Reduced eosinophilic fibrous tissue and increased pale basophilic soft tissue. (Other choices: Extensive calcification, Amyloidosis w/apple green birefringence on polarization microscopy, Coag necrosis w/abundant eosinophilic granular debris.) Test q: During the past year, a 34F has had palpitations, fatigue, and worsening chest pain. On phys exam, she is afebrile. Her pulse is 75/min, resp 15/min, and BP 110/70 mmHg. Auscultation of the chest indicated a midsystolic click w/a late systolic murmur. A review of systems indicates that the patient has one or two anxiety attacks per month. An echocardiogram is most likely to show which of the following? Mitral valve prolapse. (Other choices: Aortic valvular vegetations, Pulmonic stenosis, Patent ductus arteriosis, Tricuspid valve regurgitation)

Complications of Floppy MV (3%) Infective endocarditis Mitral insufficiency slow onset or sudden Stroke (thrombi on atrial side) Arrhythmia or sudden death Higher risk for complications in men, older persons, and in patients who already have an arrhythmia, LV enlargement, or MV regurgitation

1. Mitral valve prolapse, from Robbins 6 ed. Note interchordal hooding. Leaflets of mitral valve ballooning upward. During systole, this valve should be fairly flat. Stretchy, mucoid/myxoid valvular tissue. See stretching of the chordae tendenae can even lead to them snapping. 2. Mitral valve prolapse, note long, slender chordae. Fluffy, upward-bulging, thick whitish glistening valvular tissue. See intrachordal hooding 3. Mitral valve prolapse, note mural thrombi in left atrium. Long, thin chordae tendineae. Bulging, thick, soft weak mitral valve. Leaflets bulged back far enough that it rubbed the left atrial wall caused friction lesion, developed mural thrombi (arrows).

th

4. Mitral valve surgically excised from a 22 month old male with Marfans, prolapse and regurgitation. Cant see intrachordal hooding but can see thickened character of the mitral valve. Ususally has delicate, thin appearance. 5. Aortic valve leaflet surgically excised from a 19M with Marfans, note thickening and myxoid change.

Rheumatic Heart Disease Rheumatic fever is an immune mediated attack on the heart after a bout of pharyngitis due to group A (beta hemolytic) streptococcus The acute phase of RF causes a pancarditis with: o Serofibrinous (bread and butter) pericarditis o Myocarditis o Valvulitis, mainly of left-sided valves
Test q: Infection with what organism precedes rheumatic fever? Beta hemolytic streptococcus. (S. aureus, N. meningitidis, Cardiobacterium) Test q: Concerning the anatomic distribution and histologic appearance of rheumatic heart disease, which of the following is true? It is a pancarditis, meaning that all 3 layers of the heart may be involved. (Other choices: The histologic abnormalities are limited to the myocardium, The right-sided valves are more severely involved than the left-sided valves, Granulomatous inflammation in the pericardium is typical.)

Notes: Rheumatic disease is an immune mediated inflammatory disease that occurs about 3 weeks (range 1 to 5 weeks) after a bout of acute pharyngitis due to group A (beta hemolytic) streptococcus (not after infections of skin or other sites; not after infections with other strep.) usually in children 5 15 years. Occurs in only about 3% of pts with group A strep pharyngitis & has been declining over 30 years (? Better treatment, ? decreased virulence of bug). Diagnosed clinically by the Jones criteria need 2 major criteria or 1 major + 2 minor along with evidence of a preceding group A strep infection: - Major Jones criteria: Migratory polyarthralgia in large joints Carditis (heart failure or pericarditis) Subcutaneous nodules Erythema marginatum of skin Sydenham chorea (never seen w/arthritis but may coexist w/carditis) Immune system attack on cerebellum jerky movements - Minor criteria: Fever, arthralgias, increased serum acute phase reactants After an initial attack of rheumatic fever, the pt is more vulnerable to a future attack & carditis is likely to worsen with each subsequent attack. Valve damage is cumulative. Histologically, granuloma-like lesions called Aschoff bodies characterize the acute phase. Aschoff bodies contain plump macrophages called Anitschkow cells, T-lymphocytes, a few plasma cells, and fibrinoid material. Aschoff bodies may be found in any layer of the heart (recall from C603 Lab). The acute valvulitis is characterized by inflammation & fibrinoid necrosis along the lines of closure, especially on the left-sided valves, with small (1 2 mm) verrucous vegetations along the lines of closure. Mitral valve regurgitation may cause plaques in the LA (jet lesion) called MacCallum plaques.
Test q: The histopathologic hallmark of acute rheumatic fever is: The Aschoff body. (Other choices: Caseating granuloma, Noncaseating granulomas w/multinucleate giant cells, Lymphocytic interstitial infiltrates in the myocardium w/myocyte injury, There is no specific histopathologic finding.) Test q: A 12M was brought to the physician w/a sore throat and fever 3wk ago and a throat culture was positive for group A beta-hemolytic Streptococcus. On the follow-up exam, the child is afebrile. His pulse is 85/min, resp 18/min, and BP 90/50. On auscultation, a murmur of mitral regurgitation is audible and there are diffuse rales over both lungs. The child is admitted to the hospital, and over the next 2 days has several episodes of atrial fibrillation accompanied by signs of acute LV failure. Which of the following pathologic changes occurring in this childs heart during hospitalization is most likely to be the cause of the LV failure? Myocarditis. (Other choices: Amyloidosis, Endocardial fibroelastosis, Fibrinous pericarditis, Tamponade.) Test q: A 10F develops subcutaneous nodules over the skin of her arms and torso 3wk after a bout of acute pharyngitis. She manifests choreiform movements and begins to complain of pain in her knees and hips, particularly with movement. A friction rub is heard on auscultation of the chest. Which of the following serum lab findings is most characteristic of the disease affecting this patient? Elevated anti-streptolysin O level. (Other choices: Elevated cardiac troponin I level, Positive ANA test, Elevated creatinine level, Positive rapid plasma regain test.)

1. Mitral valve vegetations in acute rheumatic fever. Valvulitis involving mitral valve. Destructive vegetations along closure lines of mitral valve. Amorphous cluster of fibrin, maybe a few inflammatory cells. 2-3. Aschoff body (slide #63 from C603 slide set) Note caterpillar cells with grooved nuclei. Not like a classic granuloma these are basement membrane-derived cells (related to histiocytes). Elongated, zig-zag nuclei.

Chronic Rheumatic Disease Involves the valves not the pericardium or myocardium o Valves involved in the order of their closing pressure. Is due to post-inflammatory thickening and deformation of the leaflets Involves: o Mitral valve virtually always (65 70% alone) o MV + AV if two valves o MV + AV + TV if three valves Four valve involvement rare
Test q: Cardiac valve disease due to chronic rheumatic disease most commonly involves only one valve. Which valve is most commonly involved? Mitral. REPEATED x3 Test q: Cardiac valve disease due to chronic rheumatic disease most commonly involves only one valve. It may occasionally involve two valves. If two valves are involved, which two are most likely to be involved? Mitral and aortic. (Other choices: Aortic/tricuspid, Pulmonary/tricuspid, Tricuspid/pulmonic) Test q: If chronic rheumatic heart disease involves only two cardiac valves, these are most likely to be: The left sided valves (mitral and aortic).

Basic Features of Chronic Rheumatic Heart Disease Fibrous thickening of the valve leaflets with fusion of the commissures button hole or fish mouth appearance Thickening, shortening and fusion of the chordae tendineae LA may dilate & harbor thrombus LV is protected if MV only May eventually get RV hypertrophy Note: Microscopic of valves in chronic heart disease shows fibrosis with vascularization and loss of the normal lamination. Rarely, Aschoff bodies may be seen.
Test q: Your gross exam of an autopsy heart from an adult shows thickening of the mitral valve leaflets w/fusion of the commissures. The chordae tendineae are short and thick. The heart shows some RV hypertrophy and dilatation of the left atrium. The other cardiac valves are normal. The most likely dx is: Chronic rheumatic valve disease. (Other choices: Healed infectious endocarditis involving the mitral valve, Floppy mitral valve mitral valve prolapse, Congenital dysplasia of the mitral valve, Cannot make the dx from this info) Test q: An elderly female had a history of repeated bouts of pulmonary edema due to mitral stenosis. At autopsy, her mitral valve showed fibrous thickening of the leaflets, fusion of the leaflets at the commissures, and shortening and thickening of the chordae tendineae. The left atrium was dilated, but there was no LV hypertrophy, and the other valves were unremarkable. The most likely diagnosis is: Chronic rheumatic disease. (Other choices: Floppy mitral valve myxoid degeneration of the mitral valve, Healed bacterial valvulitis, Cannot make a diagnosis from this info, Acute rheumatic fever)

1. Chronic rheumatic mitral valve with acute vegetations. View looking down from LA onto the mitral annulus. Fishmouth, button-hole appearance. Commisures tend to be fused (where one leaflet joins another) fairly unique to advanced rheumatic disease. 2. Chronic rheumatic mitral valve viewed from above, note fusion of commissures on both sides. 3. Chronic mitral valve showing shortening and thickening of the chordae and vascularization of the leaflets. Adhesion/fusion of chordae. Scar tissue shrinkage of chordae.

4. Chronic rheumatic disease of the aortic valve, note fusion of the commissures. 5. Chronic rheumatic mitral valve viewed from above. Mitral valve thickened, pillow-looking. 6. Chronic rheumatic mitral valve showing shortening, thickening and fusion of the chordae. Also see some thrombus, injury to LA.

Rheumatic Heart Disease Pearls The severity of joint involvement is inversely proportional to heart involvement In acute RF, heart involvement is typically not symptomatic unless there is heart failure or pericarditis Chronic rheumatic heart disease is typically clinically silent for many years after acute RF The long term prognosis is highly variable prosthetic MV replacement may be required Mitral regurgitation may develop quickly (due to restricted mobility of leaflets or ventricular dilatation), but stenosis takes years Chorea is a late manifestation of RF, male > female o May last from a week to >2 years! o Chorea is never seen with arthritis, but may coexist with carditis o Hypotonia and emotional disturbances are typical Subcutaneous nodules attached to tendon sheaths extensor surfaces & over bony prominences of upper & lower extremities & mastoid. Histologically Aschoff bodies Erythema marginatum red margins progress as center clears see on trunk and proximal limbs Death in acute RF is rare, and is usually due to myocardial involvement (heart failure RF and post-streptococcal glomerulonephritis rarely coexist Infective Endocarditis Infection of the heart valves or other endocardial surfaces by a wide variety of bacterial and fungal organisms Basic lesion is the vegetation Can be divided by clinical and pathologic features into acute and subacute types Acute Endocarditis Rapidly destroys heart valves & has a high mortality ~ 50% Virulent organisms; example Staph. aureus May affect normal valves Rapid development of chills, fever May rapidly destroy valve insufficiency May ring abscess perforation of heart Large vegetations common, may embolize May cause acute glomerulonephritis due to Ag-Ab complexes deposited in glomeruli Note: Embolic complications: If right-sided, may embolize to lungs & cause abscesses. If left-sided, may embolize to brain, kidneys or other sites & cause abscesses, or may embolize to coronary arteries & cause MI. 1. Bacterial endocarditis of a bicuspid aortic valve due to Staph. aureus. Destruction of cusps and ring abscess formation. 2. Infective endocarditis of the aortic valve due to a virulent organism. Acute infection. Very destructive to valve leaflets large vegetations. Subacute Endocarditis May have a long course weeks to months Lower virulence organisms; example Strep. viridans More likely to affect abnormal structures or prosthetic devices o CHD, bicuspid aortic valve, floppy mitral valve, etc. Smaller vegetations, less destructive than acute May cause fever of unknown origin Clinical features range from flu-like symptoms, fever, wt loss, murmur especially if left-sided o Compare to acute extremely ill/toxic presentation Most can be successfully treated with antibiotics Note: Subungual hemorrhages, petechiae and Roth spots in the eyes may be seen with subacute bacterial endocarditis secondary to embolized bits of vegetations.

Bugs That Cause Infectious Endocarditis Staph. aureus 10 to 20% of cases, can attack normal or abnormal valves, highly virulent Strep. viridans (alpha hemolytic strep.) can attack damaged native valves Other bugs include Haemophilus, Actinobacillus, Cardiobacterium, Eikenella & Kingella (HACEK group) Many others can cause, including GNRs and fungi About 10% of cases are blood culture negative
Test q: A 60M presents w/fever and malaise. Petechial hemorrhages are noted on the skin and in the nail beds. A skin punch biopsy of one of the skin lesions shows a neutrophil infiltrate surrounding small blood vessels w/hemorrhage into the dermis. Echocardiography shows a probable vegetation. Assuming the patient has no previous anatomic abnormalities of the cardiac valves, which of the following organisms is the most likely infectious agent? S. aureus. (Other choices: Cardiobacterium species, Group A beta hemolytic strep, Any organism from the HACEK group, Candida.) REPEATED x2 Test q: Which of the following organisms is most likely to infect an anatomically normal cardiac valve? S. aureus. (Other choices: Cardiobacterium species, Group A beta hemolytic strep, Any organism from the HACEK group, Fungal organisms are most likely to do this.) Test q: A rapidly destructive infection of a previously normal aortic valve that results in a ring abscess (an abscess involving the aortic valve annulus) is most likely caused by which of the following organisms: S. aureus. (Other choices: Strep viridans, Cardiobacterium species, Actinobacillus species, Any member of the HACEK group.) Test q: A young adult IV drug abuser w/previously normal cardiac anatomy developed a rapidly destructive infection of the aortic valve. This infection resulted in a ring abscess (invasion of the tissue around the annulus of the valve) and the abscess subsequently perforated the heart into the pericardial sac. The most likely causative organism is: S. aureus. (Other choices: Cardiobacterium hominis, S. viridans, Group A beta hemolytic strep) Test q: Disruption of the integrity of the aortic valve w/resulting insufficiency may be a complication of all of the following EXCEPT: Atherosclerosis of the ascending aorta. (Other choices: Marfans, Syphilitic/luetic aortitis, Aortic dissection, S. aureus infection of the aortic valve.)

Infectious Endocarditis Predisposing Lesions Any anatomic cardiovascular lesion predisposes to IE. Examples: o Bicuspid aortic valve o Any type of congenital heart disease o Calcific aortic stenosis; calcified mitral annulus o Rheumatic valve disease o Prosthetic valves o Previously damaged native valves o Floppy mitral valve Note: Dentists and cardiologists have long known of the need for prophylactic antibiotics with any dental or surgical procedure that may cause bacteremia in patients with predisposing anatomic lesions.
Test q: Floppy mitral valve, calcific aortic stenosis, and bicuspid aortic valve share: An increased likelihood of valvular infection resulting from bacteremia. (Other choices: Atherosclerosis as the underlying cause, Myxoid degeneration of connective tissue as the underlying cause, An association w/aortic dissection, These lesions have nothing in common whatsoever.)

Two Types of Non-infectious Valvular Vegetations Non-bacterial thrombotic endocarditis (marantic endocarditis) occurs in extremely debilitated patients (cancer, sepsis) Endocarditis of systemic lupus erythematosus (Libman-Sacks endocarditis) Notes: Marantic endocarditis probably results from a hypercoagulable state such as disseminated intravascular coagulation (DIC) or some forms of cancer (can be part of Trousseau syndrome). The vegetations are sterile, small and do not damage valves, but can embolize. The vegetations form on normal valves. Microscopically there is no inflammation. SLE vegetations are small and sterile, and frequently form on the undersides of AV valves. They may be seen on the endocardium and chordae too. Microscopically there may be intense inflammation and fibrinoid necrosis involving the leaflet. Scarring of the valves can occur.
Test q: All of the following are true of marantic endocarditis EXCEPT: The lesions, if cultured, yield organisms such as S. viridans and the HACEK group organisms. (Other choices: It may result from disseminated intravascular coagulation, It may occur on structurally normal valves, It may be associated w/cancer.) Test q: Which of the following conditions may complicate rheumatic fever, systemic lupus erythematosus, and MI? Immunologically mediated pericarditis. (Other: Dressler syndrome, Aortic dissection, Amyloidosis, Toxin-mediated myocarditis) Test q: A 45F present w/chest pain and fever, and on phys exam a cardiac friction rub is detected. A pericardial effusion (150mL estimated volume) is noted radiographically. Patient history of which of the following conditions would be most likely to explain this presentation? Systemic lupus erythematosus. (Other choices: MS, COPD, Idiopathic inflammatory bowel disease, Primary sclerosing cholangitis)

1. Bacterial endocarditis of mitral valve due to Strep. viridans. Smaller in size, less destructive in nature. Less likely to embolize. 2. Infective endocarditis dark blue clumps of bacteria are seen on H&E stain. Varying amounts of fibrin in vegetations. Bacteria can be w/calcification or mixture of inflammatory cells. 3. 13M with ventricular septal defect (VSD) involving the infundibular septum, viewed from left ventricular side. S. aureus infection. 4. Same heart as #3: Wood stick through VSD, note mural thrombus on opposite wall of right ventricular outflow tract. Flip the heart around can see the lesion located in the infundibulum. Get erosion of the endothelium as hole gets smaller, stream gets faster rips endothelium seeded w/S. aureus. 5. Multiple lung abscesses due to infected emboli from right heart, patient died of acute pulmonary hemorrhage. Diseases of the Myocardium General categories: Myocarditis (including transplant rejection) Cardiomyopathy (noninflammatory myocardial dysfunction of unknown [now sometimes known] cause) Other (cardiac tumors, metabolic and storage diseases, toxic injury) Note: There is some overlap. Old, burned out viral myocarditis may become dilated cardiomyopathy. A metabolic disease of the myocardium may also, in a sense, be considered a cardiomyopathy. Myocarditis Refers to inflammation of the heart which injures cardiac myocytes May be infectious or noninfectious: o Viruses, Chlamydia, Rickettsiae, bacteria, fungi, protozoa, helminths Viruses are most common north of the equator. o Rheumatic fever, postviral, lupus, drug hypersensitivity o Heart transplant rejection o Autoimmune aka lymphocytic, secondary to a viral infection elsewhere o Unknown cause (sarcoidosis, giant cell myocarditis) th Note: See Table 13-10, p. 584, Robbins 6 Ed. for major causes of myocarditis. Viral Myocarditis The most common cause of myocarditis in the US is viral, usually due to coxsackieviruses A and B and other enteroviruses These are very difficult to isolate by viral culture of a myocardial biopsy; PCR may help The inflammatory injury in viral myocarditis may be secondary to an immune cross reaction with myocytes and not by direct viral cytopathic injury Figure: Typical histology of viral myocarditis or acute transplant rejection infiltrate is mainly lymphocytic

Viral Myocarditis (Clinical) May be asymptomatic or minimally symptomatic May rapidly develop heart failure, arrhythmias, or may cause sudden death There may be a murmur due to cardiac dilatation causing mitral regurgitation May heal completely, or may progress to dilated cardiomyopathy years later Notes: Grossly, hearts with active myocarditis are described as soft and flabby. The myocardium may be mottled and the heart may be dilated. Microscopically, viral myocarditis is characterized by a (nonspecific) lymphocytic infiltrate with myocyte necrosis. Figure: Death from acute myocarditis in a 13F with a 3-day history of flu-like symptoms. LV looks dilated, myocardium mottled/congested. Would feel soft, flabby.

Test q: The most common cause of infectious myocarditis in the US is thought to be: Coxsackie B viral infection. (Other choices: S. aureus, Cardiobacterium species, Trypanosoma cruzi, Toxoplasmosis, CMV) REPEATED x2 Test q: A 32y/o North American Caucasian female presents w/increasingly severe symptoms of CHF over 2wk. She has no previous history of heart disease. An electrocardiogram shows some runs of ventricular tachycardia, and an endomyocardial biopsy shows focal myocyte necrosis along w/a lymphocytic infiltrate. She is most likely to have an infection with which of the following organisms? Coxsackievirus A. (Other choices: T. cruzi, S. viridans, Toxoplasma gondii, S. aureus.)

Chagasic Myocarditis Chagas disease the protozoan bug is Trypanosoma cruzi Important in South America 80% with heart involvement may be lethal or may go into chronic phase Transmitted by insects such as reduvid bug Microscopically see parasites within myocytes and mixed inflammation Note: T. Cruzi may also cause destruction of the myenteric plexus of the esophagus, duodenum, colon and ureter with subsequent dilatation of these viscera (megacolon).

Test q: The photograph shows myocardium from a 5F who died suddenly at home. There was no trauma and no significant history of chronic health problems. A history of diarrhea several days prior to death was obtained. A likely etiologic agent is: Coxsackie virus. (Other choices: Treponema pallidum, Cardiobacterium hominis, Mycobacterium tuberculosis, CMV.) (Similar picture to the original 2009 Exam 1 #34)

Giant Cell Myocarditis Has a rapid onset with a poor prognosis Microscopically see myocardial necrosis with lymphocytes, plasma cells, eosinophils and giant cells Has some overlap with sarcoidosis, but sarcoidosis has a ! more insidious onset Some patients may recover completely or with some myocardial fibrosis Etiology is unknown Hypersensitivity Myocarditis Due to hypersensitivity reaction to drugs such as methyldopa, sulfonamides Microscopically see interstitial inflammatory infiltrates, mainly perivascular, with lymphocytes, macrophages, and lots of eosinophils Some Causes of Metabolic Myocardial Injury Iron overload (hemochromatosis) Hyperthyroidism tachycardia, palpitations, cardiomegaly, sometimes arrhythmias Hypothyroidism stroke volume & rate, sluggish flow, may get myxedema of heart Disorders of energy use (carnitine def, FOX disorders, mitochondrial disorders) Notes: Iron accumulates in cardiac myocytes and interferes with metal dependent enzyme systems. Hypothyroidism may cause myxedema of the heart with myofiber swelling, basophilic change, loss of cross striations, and increased mucopolysaccharide content of the interstitium. Some disorders of energy metabolism can cause death in infancy and be confused with SIDS (example LCHAD deficiency).

Cardiomyopathy Originally defined as a primary (intrinsic) abnormality of the myocardium Cause unknown (not always true now) Excludes so-called ischemic CM o Seen in pts w/atherosclerotic cardiovascular disease, perhaps w/repeated MI. Result of remodeling of heart, increase in size of heart due to chronic ischemia (tries to compensate for loss of myocytes). Different than ischemia in other organs ex: kidney atrophies. CM may be divided into three classic clinical/functional/pathologic patterns: o Dilated CM most common ~ 90% Rounded, enlarged especially LV o Hypertrophic CM Not much increase in total size of heart, so hard to detect on CXR. Weight of heart increases. Most common cause of sudden death in young athletes. o Restrictive rarest Shape may be normal. Characterized by stiffening of myocardium, especially the LV. Less able to expand on diastole, less able to contract on systole. Classic cause is amyloidosis. Dilated Cardiomyopathy Features a rounded, dilated heart (esp LV) which also usually is hypertrophied The heart is hypocontractile CHF Poor prognosis, 50% mortality in 2 years May affect any age May be treated by cardiac transplant A variant is arrhythmogenic right ventricular dysplasia causes heart failure and may cause sudden death in young persons o Fibrofatty infiltration of RV myocardium Note: Dilated CM is also known as congestive cardiomyopathy. Dilated CM may cause death by heart failure or arrhythmia.
Test q: A teenage athlete died suddenly during a game. Autopsy exam of the heart showed dilatation of the RV w/nearly transmural replacement of the RV free wall by fat and fibrosis. The most likely diagnosis is: Arrythmogenic right ventricular dysplasia. (Other choices: Dilated cardiomyopathy, Hypertrophic cardiomyopathy, Myocarditis, Restrictive cardiomyopathy.)

Causes of Dilated CM Burned out myocarditis EtOH or other toxicity Peripartum CM occurs late in pregnancy or several weeks to months postpartum poorly understood Genetic 2- 30% of cases, variable inheritance, mostly autosomal dominant, & variable defects - one is abnormal cytoskeletal (NOT CONTRACTILE) protein like Duchenne & Becker muscular dystrophy Mostly the cause is unknown
Test q: Dilated cardiomyopathy may be associated with each of the following EXCEPT: Mutations involving contractile proteins. (Other choices: Alcohol toxicity, A history of myocarditis, Muscular dystrophy) Test q: What clinicopathologic features do aortic dissection and dilated cardiomyopathy have in common? Either may occur during pregnancy for poorly understood reasons. (Other choices: Both are caused by hypertension, Both are associated w/Marfan syndrome, Neither are associated w/genetic defects, These entities have absolutely nothing in common.) Test q: Dilated cardiomyopathy and aortic dissection share a known association with: Pregnancy or the postpartum state. (Other choices: Marfan syndrome, Vit E deficiency, Hypertension, Atherosclerosis)

Notes (Dilated CM): The progression from viral myocarditis to dilated cardiomyopathy has been demonstrated by serial myocardial biopsies. However, in most cases of DCM, the heart shows little or no residual inflammation. Toxic causes of DCM are hard to prove no good way to tell apart from other DCM. The etiology of peripartum CM may be multifactorial, including factors such as hypertension, volume overload, poor nutrition, possible metabolic abnormalities or autoimmune phenomena.

Morphology of Dilated CM Rounded, dilated heart, 2 3x expected weight LV wall may be thinner due to dilatation May see endocardial fibrosis, may see mural thrombus in chambers Coronary arteries OK Valves OK, but possible regurg of AV valves due to annulus dilatation Histology not too exciting myocyte hypertrophy, some interstitial fibrosis, rarely some residual myocarditis
Test q: In dilated cardiomyopathy, mitral valve insufficiency may develop due to: Dilatation of the mitral annulus and failure of the annulus to adequately contract during systole. (Other choices: An abnormality of the cytoskeletal proteins of the valvular fibrocytes, Stretching and lengthening of the chordae tendineae, Development of a friction lesion along the LVOT, Thickening of the mitral valve leaflets w/fusion of the commissures and shortening and thickening of the chordae tendineae.) REPEATED x2 but one year, the answer was just Dilatation of the mitral valve annulus. Test q: What do syphilitic aortic aneurysm and dilated cardiomyopathy have in common? Both may cause dilatation of a heart valve ring w/subsequent valvular insufficiency. (Other choices: Both are caused by spirochete infections, Syphilitic aortic aneurysm typically results in a secondary dilated cardiomyopathy, The pathogenesis of both is unknown, These entities have nothing in common.) Test q: What do floppy mitral valve, rheumatic mitral valve stenosis, and dilated cardiomyopathy have in common? All may result in dilatation of the left atrium and deposition of mural thrombus within the left atrium (Other choices: All are caused by genetic abnormalities of connective tissue or cardiac myocytes, All are associated w/long thin chordae tendineae, Aschoff bodies are typically seen in all of these entities, These entities have absolutely nothing in common.)

1. Dilated cardiomyopathy, Note rounded/spherical shape of left ventricle. Very heavy, but could be normal thickness. 2. Dilated cardiomyopathy, note rounded, thin-walled left ventricle. Less effect in RV.

Hypertrophic Cardiomyopathy Has many names: o Idiopathic hypertrophic subaortic stenosis (IHSS) o Hypertrophic obstructive cardiomyopathy o Asymmetric septal hypertrophy (ASH) Features a heart that is normal in shape, but with marked wall thickening due to hypertrophy (esp LV) often with asymmetric thickening of the septum and narrowing of the LV cavity The heart is hypercontractile without LV cavity dilatation The problem is diastolic filling and LVOT obstruction Has a better prognosis than DCM, but may cause angina, CHF, A-fib, mural thrombus, infective endocarditis of MV Can affect children or adults HCM is one of the most common causes of sudden death in children, adolescents, and young athletes
Test q: The most significant pathophysiologic problems in hypertrophic cardiomyopathy is (are): Poor diastolic filling of the left ventricle and left ventricular outflow tract obstruction. (Other choices: A hypocontractile heart and mitral regurgitation, Coronary artery disease, Dilatation of the mitral valve ring w/subsequent mitral insufficiency, Secondary pulmonary hypertension.) Test q: The basic physiological abnormality of the heart w/hypertrophic cardiomyopathy is: Poor filling of the LV during diastole. (Other choices: Insufficient ability of the hypertrophied heart to contract, Stretching of the cardiac myocytes due to an abnormality of cytoskeletal proteins, The basic abnormality is in the conducting system, Heart w/hypertrophic cardiomyopathy are actually physiologically normal.) Test q: A teenage basketball player collapsed and died suddenly during a game. The young man had reportedly been asymptomatic and with no history of hypertension. The autopsy shows a 700g heart that was not markedly enlarged on external appearance. His coronary arteries were normal. A complete autopsy, including toxicology studies, was otherwise negative. The most likely cardiac diagnosis is: Hypertrophic cardiomyopathy. (Other choices: Dilated cardiomyopathy, Arrhythmogenic RV dysplasia, Restrictive cardiomyopathy, The cause of death was probably not cardiac.)

Notes (Hypertrophic CM): The septum may bulge into the left ventricular tract. This acts in concert with the adjacent anterior leaflet of the mitral valve to produce left ventricular outflow tract obstruction. A harsh systolic ejection murmur may be present. Late cardiac dilatation in the course of HCM may actually relieve the LVOT obstruction. To diagnose HCM, it is necessary to rule out other causes of myocardial hypertrophy such as hypertension and aortic stenosis. Sudden death may be due to blockage of the left ventricular outflow by the anterior leaflet of the mitral valve. The leaflet may be sucked into the LVOT by venturi action of the blood flowing through the narrowed passage. HCM can be managed medically better than DCM. Causes of Hypertrophic CM Has a genetic basis in all cases. Most are familial and typically autosomal dominant with variable expression; the rest are sporadic Often genetic errors are in coding for contractile proteins; >100 mutations known o Remember, dilated CM involves defective cytoskeletal proteins Investigation of family members of patients with HCM is indicated, since about half of cases are familial. Morphology of Hypertrophic CM Thickened LV wall at the expense of the cavity, often with asymmetric septal thickening and bulge of septum into LVOT Heart may be normal in external shape (may look normal on a routine chest X-ray), but 2-3x expected weight Anterior leaflet of MV is thickened, and there may be a friction lesion on the LVOT Histology shows myocyte hypertrophy, some interstitial fibrosis, and sometimes myofiber disarray, especially in septum Note: Myofibrillar disarray within cardiac myocytes in HCM can be seen with electron microscopy.
Test q: At autopsy, a 35M has cardiomegaly and a markedly thickened LV septum. The thickening is asymmetric (meaning the septum is thickened much more than LV free wall). Which of the following is a likely diagnosis? Hypertrophic cardiomyopathy. (Other choices: Duchenne MD, Marfans syndrome, Dilated cardiomyopathy secondary to adriamycin treatment, Cardiac hypertrophy secondary to hypertension) Test q: A 10F who is normally developed has chronic progressive exercise intolerance. Phys exam shows temp 37.1C, pulse 70/min, resp 14/min, and BP 100/60. A CXR shows cardiomegaly and mild pulmonary edema. An echocardiogram shows severe LV hypertrophy and a prominent IV septum. The RV is slightly thickened. During systole, the anterior leaflet of the mitral valves moves into the outflow tract of the LV. The ejection fraction is abnormally high, and the ventricular volume and cardiac output are both low. Which of the following is the most likely cause of the cardiac abnormalities in this patient? Mutation in the -myosin heavy chain. (Other choices: Autoimmunity against myocardial fibers, Excessive iron accumulation, Deposition of amyloid protein, Latent enterovirus infection) REPEATED x3

1. Hypertrophic CM note thickened LV wall, especially septum, with narrowed LV outflow tract, and fiber disarray on trichrome stain (red = cardiac myocytes; blue = fibrous tissue). See pencil-thin LV cavity. Often, the septum gets thicker than the free wall ASH (asymmetric septal hypertrophy) can bulge into the LVOT, markedly narrows it. Anterior leaflet of mitral valve gets sucked up into LVOT, ends up rubbing the opposite side on the septal aspect of the LVOT. After all the rubbing (every single heartbeat), eventually results in a thickened fibrous whitish plaque on LVOT (often in the shape of the mitral valve leaflet). LVOT obstruction may be the cause of death in some athletes. 2. Hypertrophic CM note thick ventricular septum with leiomyoma-like appearance indicating fiber disarray. Have no cardiac reserve, so cannot compensate for strenuous physical activity because of the small LV chamber.

Restrictive Cardiomyopathy This is a disorder of ventricular compliance The heart is normal in shape but too stiff Constrictive pericarditis is clinically similar heart is mechanically prevented from contracting well Causes of Restrictive CM Things that infiltrate and stiffen the myocardium: o Amyloidosis o Sarcoidosis o Metastatic tumor o Some storage diseases o Endomyocardial fibrosis (a tropical disease of unknown cause) Things that thicken and stiffen the myocardium and endocardium: o Endocardial fibroelastosis (in children - occurs with some types of congenital heart disease and rarely occurs alone) o Loeffler endomyocarditis (assoc. with eosinophilia or eosinophilic leukemia) o Endomyocardial fibrosis (a tropical disease of unknown cause) Idiopathic restrictive CM myocardium shows patchy or diffuse fibrosis Note: In RCM the heart is about normal size and the myocardium is firm (waxy in amyloidosis).
Test q: What do amyloidosis of the myocardium, sarcoidosis of the myocardium, and endomyocardial fibrosis have in common? All may cause a restrictive pattern of cardiomyopathy. (Other choices: All may cause dilated cardiomyopathy, All may cause hypertrophic cardiomyopathy, All are due to deposits of abnormal protein within the interstitium of the myocardium, Aschoff bodies are typically seen in all of these entities.) Test q: Restrictive cardiomyopathy w/impaired filling of the left ventricle due to a stiffened myocardium is most likely to result from which of the following? Cardiac amyloidosis. (Other choices: Dilated cardiomyopathy, Ischemic cardiomyopathy, Viral myocarditis, Marfans syndrome) Test q: An 86M has had increasing dyspnea and reduced exercise tolerance for the past 7yr. On phys exam, he is afebrile and has a BP of 135/85. An irregularly irregular HR averaging 76/min is audible on auscultation of the chest. Crackles are heard at the bases of the lungs. A CXR shows mild cardiomegaly and mild pulmonary edema. Echocardiography shows slight right and left ventricular wall motion, reduced LV filling, and an ej fraction estimated to be 25%. An endomyocardial biopsy specimen shows amorphous pink-staining deposits between myocardial fibers but no inflammation and no necrosis. Which of the following is the most likely diagnosis? Cardiac amyloidosis. (Other choices: Rheumatic heart disease, Constrictive pericarditis, Mitral valve prolapse, LV aneurysm) Test q: In the ER, 45min after a motor vehicle accident, a 30M who initially seemed oriented and alert becomes obtunded and hypotensive. A CXR shows an enlarged cardiac silhouette and ultrasound exam shows a large amt of fluid in the pericardial space. A pericardiocentesis is performed and 400mL of bloody fluid is withdrawn and the patients condition improves. The patients hypotension can be explained by: Impairment of cardiac filling. (Other choices: Inflammation of the pericardium, Necrosis of myocardium due to blunt trauma to the chest, Blockage of blood flow through or increased resistance in the ascending aortic arch, A cardiac arrhythmia)

Cardiac Transplantation ~2500/year worldwide; ~5 in 2009 at Riley Acute cellular rejection is a form of lymphocytic myocarditis controllable with drugs Transplant arteriopathy is a long term complication (aka graft vascular disease, graft arteriosclerosis) years to a decade or more may cause sudden death Note: Acute cellular rejection is cell mediated and is controllable by drugs such as cyclosporine. Transplant vasculopathy on the other hand, may be mediated more by humoral means, and as yet is not controllable by drugs.
Test q: A 70M who has a cardiac transplant 4 years ago has a sudden decline in cardiac output and acute cellular rejection is suspected. What findings on an endomyocardial biopsy would confirm this impression? A myocardial lymphocytic infiltrate. (Other choices: Obliteration of the small blood vessels in the myocardium, Presence of Congo red stain-positive eosinophilic material in the myocardium, The presence of neutrophils in the endocardium) REPEATED x2

Congenital Heart Disease (CHD) Def: Abnormality of heart or great vessels present at birth (may be discovered as an adult) Usually refers to structural abnormalities such as abnormal chamber and vessel relationships, abnormal connections (holes), obstructions, absence or maldevelopment of structures, and other anatomic abnormalities Usually does not include congenital tumors, infections, cardiomyopathies Ranges from trivial (bicuspid aortic valve, persistent left superior vena cava) to lethal (absent left ventricle) If severe, causes retarded development, failure to thrive, increased susceptibility to infectious diseases in childhood, and rarely sudden cardiac death Increased risk for endocarditis (generally true for any structural abnormality, congenital or acquired; i.e. prosthetic valve) Increased risk during pregnancy for women with CHD Hyperviscosity due to polycythemia Nearly twice as many children die from CHD in the U.S. as from all forms of childhood cancer combined (Am. Heart Association, 2005)

Causes of CHD Usually unknown - probably multifactorial (may see discordance of CHD in identical twins!) Around 5% chromosomal (Trisomies, Turners coarctation of the aorta) Some (? 3%) associated with non-chromosomal single gene defects (Noonans, DiGeorge, numerous others) o DiGeorge absence of thymus/parathyroid glands see conotruncal malformations (tetralogy of Fallot, interrupted aortic arch, VSD, persistent truncus arteriosus)) May be seen in many malformation associations (VATER, polysplenia) o Also, sometimes asplenia o VATER = vertebral anomolies, anal atresia, tracheoesophageal fistula, esophageal atresia, radial/renal abnormalities, cardiovascular anomalies Rare - infections (rubella) o Rubella CHD, cataracts, deafness Major CHD Defects Can Be Lumped into 3 Categories Cyanotic - blood from the right side of the heart (deoxygenated blood) enters the left side (right to left shunt) Noncyanotic - blood from the left side of the heart enters the right side (left to right shunt) Other (i.e. no shunts, example - obstructions) Right to Left Shunts Cause cyanosis from early infancy May be complicated by paradoxical emboli Left to Right Shunts No (initial) cyanosis Causes pressure (or volume) overload of the pulmonary circulation Eventually can reverse shunt direction and become cyanotic

Test q: A 20F w/congenital heart disease develops pulmonary hypertension. The development of pulmonary hypertension is most likely a complication of: A congenital heart defect w/a left-to-right shunt. (Other choices: A bicuspid aortic valve, A congenital heart defect w/a right-to-left shunt, Tricuspid atresia, Transposition of the great vessels) Test q: The development of irreversible pulmonary hypertension is a serious complication of: A congenital heart defect w/a left-to-right shunt. (Other choices: A congenital heart defect with a right-to-left shunt, All types of cyanotic congenital heart disease, Tetralogy of Fallot, Pulm hypertension does not occur as a complication of congenital heart disease.) Test q: Congenital heart defects that result in the shunting of left-sided (oxygenated systemic) blood to the right (pulmonary circulation): May cause permanent injury to the pulmonary vasculature after a period of time. (Other choices: Cause cyanosis from birth, Must be surgically corrected in all cases immediately after birth, Usually involve lesions that start w/a T, May be alleviated by surgical placement of shunt conduit between the aorta and the pulmonic trunk to relieve high pulmonic pressures.)

Eisenmengers Syndrome Is a switch from noncyanotic to cyanotic CHD due to reversal of shunt flow Due to right ventricular hypertrophy and pulmonary vascular changes after prolonged pulmonary HTN Too late to surgically repair since pulmonary vascular changes are irreversible Not the same as Eisenmengers complex
Test q: Eisenmengers syndrome is best described as congenital heart disease with which of the following? A switch from noncyanotic disease to cyanotic disease. (Other choices: Electrical conduction abnormalities resulting in arrhythmia, An associated patent ductus arteriosis, A switch from cyanotic disease to noncyanotic disease)

CHD with Right to Left Shunt (Cyanotic) Tricuspid Atresia: Tricuspid annulus fails to canalize no patency of tricuspid valve. Goes along w/obligatory ASD mixing of RL blood. TAPVR (TAPVC): Totally anomolous pulmonary venous return instead of going to LA, goes somewhere to the systemic venous system TOGV: Transposition of great vessels aortic and pulmonic trunk reversed Truncus Arteriosus: Single aortic trunk allows mixing of R & L blood Tetralogy of Fallot (if sufficient pulmonary outflow restriction) Hypoplastic left heart syndrome (probably should be considered as cyanotic): LV doesnt develop have single ventricle heart, R&L sides must mix Note: All the cyanotic types start with a T except the last one.

Test q: A newborn infant with congenital heart disease is cyanotic, even with supplementary oxygen. The category of his heart disease is most likely: A shunt defect with right-to-left flow. (Other choices: A shunt defect with left-to-right flow, An obstructive defect such as congenital aortic stenosis, A non-obstructive non-shunt defect such as an aberrant course of a coronary artery, A large VSD.) REPEATED x2 Test q: The differential diagnosis for a cyanotic newborn w/a suspected cardiovascular abnormality would include all of the following except: Ventricular septal defect. (Other choices: Tricuspid atresia, Transposition of the great vessels, Truncus arteriosus)

CHD with Left to Right Shunt (Noncyanotic) ASD (increased pulmonary blood volume more than pressure) VSD AV septal Defect (AV canal) seen in Down syndrome PDA Note: All the noncyanotic types have a D.
Test q: Which of the following congenital cardiac abnormalities is typically noncyanotic? Atrial septal defect. (Other choices: Transposition of the great vessels, Tetralogy of Fallot, Truncus arteriosus) Test q: The differential diagnosis for a non-cyanotic newborn w/a suspected congenital cardiovascular abnormality would include all of the following except: Tetralogy of Fallot. (Other choices: Atrial septal defect, Patent ductus arteriosus, Ventricular septal defect) REPEATED x2 Test q: Based on the gross photo, the most likely abnormality in this autopsy heart from a newborn is: Patent ductus arteriosus. (Other choices: Truncus arteriosus, Hypoplastic left heart syndrome, Tetralogy of Fallot.) Could not find this picture online it is 2007 exam 2, #31.

Atrial Septal Defect (ASD) 90% are secundum (at foramen ovale) - the rest are rare Not the same as patent FO (present in up to 1/3 of normals) th FO 5 valve of the heart. In the fetus, allows passage of blood through the atrial septum into the left side of the heart so the left heart can develop. This rich blood is preferentially pumped up to the head/brain so it can develop. After clipping of umbilical cord, dont need FO anymore. Lung vascular beds open up, and pressure on the left becomes greater flap valve closes. Eventually, in most of us it fuses and becomes continuous septum. In 1/3 of people, is patent and can open up if R pressures become higher than L. Can lead to paradoxical emboli. Shunts left to right (noncyanotic) May be asymptomatic for a long time (30 yo) or a life time Increased lung volume of flow more than pressure May eventually get volume hypertrophy of right ventricle, <10% develop irreversible pulmonary HTN o If a patient has a big ASD and a murmur, its probably a pulm ejection murmur due to greatly increased flow in right heart. Paradoxical embolism: A type of stroke or other form of arterial thrombosis caused by embolism of a thrombus of venous origin through a lateral opening in the heart, such as a patent foramen ovale. The opening is typically an atrial septal defect.

Note: Not many examples of pure ASD in our collection


Test q: A 58F developed thromboses of the large veins of the legs during a long airline flight. During treatment of the deep venous thrombosis, the patient suffered a cerebral infarct (stroke). A possible cause of this paradoxical embolization is: An atrial septal defect. (Other choices: There is an undiagnosed congenital VSD w/a left-to-right shunt, The blood clot traveled from the deep veins of the legs through the lungs to the brain, Coarctation of the aorta.)

Figures: Atrial Septal Defects: Secundum (1), Primum (2), Sinus Venosus (3), Coronary sinus (4). Secundum = most common ASD, usually arises from an enlarged FO or inadequate growth of the septum secundum. Primum = AV septal defect. AV septal defect (aka endocardial cushion defect) involves both atria and both ventricles of the heart, creating scooped out defect in the middle of the heart have elements of both atrial and ventricular septal defects. Complete atrioventricular septal defect = single chamber heart, single AV valve separating the atrium from the ventricle. Small muscular part of IV septum remains. Associated w/Down syndrome.

Ventricular Septal Defect (VSD) Most common cardiac anomaly Frequently associated with other cardiac defects - about 30% isolated Shunts left to right (noncyanotic) initially About 90% involve membranous septum, others in muscular septum (looks like Swiss cheese) or below pulmonic valve o Membranous septum the portion just subjacent to the aortic valve annulus (the rest of the septum is muscular). Smaller defects, especially those in the muscular septum, tend to close on their own. Small defects - well tolerated, may close spontaneously Large defects - usually dont close, and over time Eisenmengers o Chronic changes occur in the vasculature of the lungs, mainly in small muscular pulmonary arterioles (due to long-standing HT, they contract to try to compensate). Become hypertrophic, media thickens, and eventually, intimal fibrosis can occur. Arteries in the lung can become elongated and tortuous (irreversible, even if VSD fixed). Can become cyanotic CHD pulm pressures can get so high that the shunt reverses and goes RL through VSD. Large defects - may have pulmonary hypertension and right ventricular hypertrophy even at birth Large subaortic defects may cause prolapse and insufficiency of aortic valve

Above: Membranous ventricular septal defect, view from left side note septal leaflet of tricuspid valve visible through defect. Most of the IV septum is muscular small triangular area in subaortic zone has fibrous membrane w/no muscle in it. Septal tricuspid leaflet slapping up against the defect can play a role in eventual closure of the VSD. Also semilunar leaflet of aortic valve looks tenuous, and with really large VSD, can get prolapse of the leaflet through the hole aortic dysfunction.

Test q: On phys exam of a non-cyanotic newborn, a murmur is detected. An echocardiogram showed a large membranous VSD (large meaning too large to close on its own). Surgical closure of the defect is planned when the infant is about a year old. This procedure: Is indicated before hypertensive pulmonary vascular disease becomes irreversible. (Other choices: Should be done immediately after birth, Is elective since the condition may be handled appropriately be medical treatment, Is technically impossible, May be postponed until middle age most patients w/this condition would not become symptomatic before that time.) REPEATED x2

Tetralogy of Fallot (TOF) Four Cardinal Features 1. RVOT obstruction (subpulmonic or valvular): Aorta shifted to the right, overrides VSD in subaortic area 2. VSD 3. Overriding Aorta 4. RV Hypertrophy boot-shaped heart Most common cyanotic CHD Sometimes ASD too (pentalogy) Basic lesion is anterosuperior displacement of infundibular septum Infundibulum: funnel/cone-shaped outflow tract of the RV lined by muscle all the way around Infundibular septum: part of the ventricular septum made up by the infundibulum May shunt either way depending on RVOT obstruction, but most are cyanotic from birth or soon after Treated initially by systemic-to-pulmonic surgical shunt (Blalock -Taussig) to direct more blood to lungs Even untreated, may survive to adulthood (if PS not too bad) May have paroxysmal hypoxemic spells (tet spells)

Above: The shunt direction in tetralogy of Fallot depends on the severity of RV outflow tract obstruction most eventually shunt R L. Mild: shunt LR (baby). See a little pulm outflow tract obstruction. Aorta overriding big VSD. Moderate: infundibular narrowing doesnt grow in proportion to the rest of the heart, baby develops relatively more pulmonary stenosis. Severe: Eventually, so much pulm stenosis that blood must go from RL. Started out pink, now cyanotic.

Test q: Classic tetralogy of Fallot is characterized by all of the following EXCEPT: Aortic stenosis. (Other choices: VSD, Overriding displaced to the right aorta, RV hypertrophy) Test q: A type of cyanotic congenital heart disease that occasionally allows survival to adulthood w/o surgical correction in infancy is: Tetralogy of Fallot. (Other choices: Tricuspid atresia, Transposition of the great arteries with normally sited ventricles and atria, Severe LV hypoplasia hypoplastic left heart syndrome, Totally anomalous pulm venous return.) Test q: An infected vegetation on the right side of the heart may result in septic emboli in the systemic circulation if: The patient has Tetralogy of Fallot (not surgically corrected). (Other choices: The patient has a small VSD w/a trivial left-to-right shunt, The fossa ovalis is anatomically sealed, The causative agent is highly virulent, It is not possible the lungs would filter out any septic embolic material from the right circulation.)

Hypoplastic Left Heart Syndrome (HLHS) Term for lesions that cause under-development (hypoplasia) of the left ventricle and ascending aorta Significant aortic valvular stenosis or atresia are the usual causes HLHS is a common indication for cardiac transplants in infants The heart appears to be all right ventricle with the left ventricle very small, sometimes slit-like The left ventricle may show thick endocardial fibroelastosis (EFE) due to high intraventricular pressure Infants are dependent on the ductus after birth, and also need an ASD to shunt left atrial blood to the right End of class notes. The following is material that was not covered in depth in the class notes but still appeared on tests. Info has been taken from the lab manual, First Aid, and Goljan Rapid Review. Coarctation of the aorta: Infantile type aortic stenosis proximal to insertion of ductus arteriosus (preductal) o INfantile IN close to the heart Adult type stenosis is distal to ductus arteriosus (postductal). Associated w/rib notching (due to collateral circulation), hypertension in upper extremities, weak pulses in lower extremities. o ADult Distal to Ductus Associated w/Turners syndrome Can result in aortic regurgitation Check femoral pulses on physical exam Most commonly associated w/bicuspid aortic valve
Test q: A 15M complains of pain in his legs when he runs more than 300m. Phys exam shows temp 36.8C, pulse 76/min, resp 22/min, and BP 165/90 mmHg. The radial pulses are 4+ and the dorsalis pedis pulses are 1+. Arterial blood gas measurement shows a normal oxygen saturation level. Which of the following lesions is most likely to be present in this patient? Coarctation of the aorta. (Other choices: Tricuspid atresia, Aortic valve stenosis, Patent ductus arteriosus, Transposition of the great arteries.) Robbins explanation: In adults, the coarctation is typically postductal, and collateral branches from the proximal aorta supply the lower extremities, leading to the pulse differential from upper to lower extremities.

Myxoma: Most common primary cardiac tumor in adults (rhabdomyoma in children) o Note: Metastasis is more common in the heart than primary tumors. 4:1 left to right atrium Ball-valve effect blocks the mitral valve orifice Fever, fatigue, malaise, anemia Complications: embolization, syncopal episodes Cardiac myxomas occur in adults and arise from the endocardium. 90% arise in the atria, more commonly on the left. Grossly, they appear as polypoid masses attached to the atrial wall.
Test q: A 49F, previously healthy, reports having suddenly lost consciousness four times in the past 6mo. In 3 instances, she was unconscious for only a few minutes. After the fourth episode 1 month ago, she was unconscious for 6hr and had weakness in her left arm and difficulty speaking. On phys exam, she is afebrile and her BP is normal. She has good carotid pulses with no bruits. Which of the following cardiac lesions is most likely to be present? Left atrial myxoma (Other choices: Pericardial effusion, Bicuspid aortic valve, Mitral valve stenosis, LAD artery thrombosis) REPEATED x3 Test q: The most common primary tumor of the heart is: Myxoma. (Other: Lipoma, Papillary fibroelastoma, Rhabdomyoma, Sarcoma) Test q: A pedunculated mass was found attached to the wall of the LA of the heart at an adult autopsy. The mass consisted of lobulated, soft, reddish, almost gelatinous tissue, and was large enough to exert a ball valve effect on the mitral orifice. The autopsy showed no evidence of neoplastic disease in any other organ or tissue. The most likely diagnosis is: Myxoma. (Other choices: Papillary fibroelastoma, Metastatic carcinoma, Lipoma, Primary malignant tumor of the heart.) Test q: A 28F w/a history of Cushings syndrome and recent fever and malaise experiences an episode of syncope. She recovers and is examined in the ER. Phys exam is significant for the presence of numerous lentigines involving the face and lips. Several 3-5mm cutaneous nodules are observed on the ear and eyelids. Serial BP measurements are highly variable. A cardiac ultrasound exam is performed and reveals a 4cm, well-circumscribed pedunculated left atrial mass. What is the most likely diagnosis for the atrial mass? Myxoma. (Other choices: Rhabdomyoma, Papillary fibroelastoma, Angiosarcoma)

Vasculitis: Classifications of vasculitis may be based on the pathenogenesis of the lesions or based on the vessels most affected. Histology: inflammatory infiltrate and other changes (necrosis, fibrin deposition, giant cells, damage to elastic fibers, hemorrhage) Difficult to recognize microscopically without clinical info. ANCA positivity is seen in many patients w/small vessel vasculitis. Pathenogenesis: Infection (syphilis, RMSF), Immunologic (SLE, Wegeners), Unknown (giant cell arteritis, PAN)

Size of vessels involved: o Large: giant cell arteritis, Takayasu arteritis o Medium: PAN, Kawasaki disease o Small: Wegeners, SLE, Henoch-Schonlein purpura, Churg-Strauss

Test q: All of the following are characteristic of the histologic appearance of vasculitis, such as that seen in Wegeners granulomatosis or polyarteritis nodosa, EXCEPT: Intravascular bacterial emboli. (Other choices: Necrosis, The presence of lymphocytes within the vessel wall and surrounding the vessel, Intravascular fibrin thrombi.)

Giant cell arteritis (temporal arteritis): Most common vasculitis that affects medium and large arteries, usually branches of carotid artery. Focal, granulomatous inflammation. Typically affects elderly females. Symptoms: Unilateral headache, jaw claudication, impaired vision (occlusion of ophthalmic artery that may lead to irreversible blindness) Findings: Associated w/an elevated ESR. Half of patients have systemic involvement and polymyalgia rheumatica. Treatment: High dose steroids
Test q: A 75M has experienced headaches for the past 2mo. On phys exam, his vital signs are: temperature 36.8 C, pulse 68/min, respirations 15/minute, and BP 130/85 mmHg. His right temporal artery is prominent, palpable, and painful to the touch. His heart rate is regular and there are no murmurs. A temporal artery biopsy is performed and the segment of temporal artery excised is grossly thickened and shows focal microscopic granulomatous inflammation. He responds well to corticosteroid therapy. Which of the following complications of this disease is most likely to occur in untreated patients? Blindness. (Other choices: Renal failure, Hemoptysis, Malignant hypertension, Gangrene of the toes) Robbins explanation: This patient has clinical features suggesting giant-cell (temporal) arteritis. This form of arteritis typically involves large to medium-sized arteries in the head (especially temporal arteries), but also vertebral and ophthalmic arteries. Involvement of the latter can lead to blindness.

Kawasaki disease: Acute, self-limiting disease of infants/kids. Strong association w/Asians. Necrotizing vasculitis of small/medium-sized vessels. May develop coronary aneurysms. Fever, congested conjunctiva, changes in lips/oral mucosa (strawberry tongue), lymphadenitis.
Test q: Autopsy of a 2y/o child who died suddenly showed massive aneurysms of the larger epicardial coronary arteries that contained thrombus. The child had been treated shortly before death for what was thought to be an infection w/swollen lymph nodes, a skin rash, and involvement of the oral mucosa. The most likely diagnosis is: Kawasaki disease. (Other choices: Takayasu arteritis, Polyarteritis nodosa, Mycotic aneurysms, A diagnosis cannot be made from this info.) Test q: Autopsy of a 14M showed large, calcified aneurysms of the proximal coronary arteries. Death was apparently due to stenosis of the distal (downstream) side of one of these aneurysms. The most likely cause of these aneurysms is: Kawasaki syndrome. (Other choices: Atherosclerosis, Syphilis, Coxsackie B viral infection, Takayasu vasculitis.) Test q: A 23M suffers an MI and dies. Autopsy demonstrated a large, recent, transmural infarct involving the anterior part of the LV and the anterior part of the IV septum. The left anterior coronary artery is markedly dilated with an aneurysm which is partially calcified. Less prominent dilation and calcification of the other coronary arteries is noted. These pathologic changes are most likely to be sequelae of which of the following disorders: Kawasaki syndrome. (Other choices: Syphilis, Coxsackie B virus infection, Giant cell arteritis.) Test q: A 3y/o child from Japan developed a fever, rash, and swelling of the hands and feet over 2 days. On phys exam, her temp is 37.8 C. There is a desquamative skin rash, oral erythema, erythema of the palms and soles, edema of the hands and feet, and cervical lymphadenopathy. The child improves after a course of IV immunoglobulin therapy. Which of the following is most likely to be a complication of this childs disease if it is left untreated? Myocardial infarction. (Other choices: Asthma, Glomerulonephritis, Intracranial hemorrhage, Pulmonary hypertension) REPEATED x2 Robbins explanation: This child has mucocutaneous lymph node syndrome, or Kawasaki disease, which involves large, medium-sized, and small arteries. Cardiovascular complications occur in 20% of cases and include thrombosis, ectasia, and aneurysm formation of coronary arteries.

Potrebbero piacerti anche